Sei sulla pagina 1di 40

‫اسئلة وزارة الصحة لاللتحاق‬

‫ببرنامج االقامة واالمتياز‬


‫( ‪) 2018 -2013‬‬

‫بفضل هللا قمت باعادة جمع وتنسيق اسئلة‬


‫الدورات السابقة‪ ,‬الصادرة من وزارة الصحة‬
‫المتحانـــات االقامـــة واالمتيــاز لالعـــوام‬
‫( ‪ , ) 2018 -2013‬بعد جهـــد وعنــاء ‪,‬‬
‫وحلها باالستعانة بالمراجع والكتب المختلفة‬
‫لتوخي الدقة في نشر المعلومة الصحيحــة‬
‫للزمالء والزميالت ‪ ,‬فإن ( احسنت فمن هللا ‪,‬‬ ‫*****‬
‫وان اسأت فمن نفسي ) ‪ ,‬واني اقـــدم هذا‬
‫العمل خالصا لوجه هللا تعالى ‪ ,‬راجيا المولى‬
‫القبول ‪ ,‬وان ينال اعجابكم ‪.‬‬ ‫‪MEDICINE‬‬

‫*****‬
‫اللهم استودعك ما قرأت وما حفظت وتعلمـت‬
‫‪ ،‬فـرده لـي عنـد حاجتـي اليه ‪ ,‬إنـــك على ما‬
‫تشاء قدير‬

‫اخوكم الدكتور ‪ :‬اسامـه الخزاعلـه‬


‫‪0799430239‬‬
‫طـــب عـــام وجراحــه‬
* Diabetes insipidus is a deficiency of what hormone * A 40 years old man presents with cold intolerance
of the following ? and weight gain . Examination reveals goiter . The
A . Atrial natriuretic peptide . most likely finding related to central nervous system
B . Vasopressin . in this patient ?
C . Aldosterone . A . Ataxia .
D . Insulin . B . Delayed relaxation of ankle jerk .
E . Progesterone . C . Hyperreflexia .
D . Hypotonia .
* An old lady presents with history of fever and left E . Loss of sensations .
sided chest pain for one month . Examination of
respiratory system shows decreased stony dull * Clostridium difficile is a nosocomial infection .
percussion note an absent breath sounds on left side . Which of the following can be used to treat it ?
Her chest X_Ray is likely A . Broad spectrum antibiotics .
to reveal ? B . PPI .
A . Collapse C . Probiotics .
B . Consolidation D . Vancomycin or metronidazole .
C . Fibrosis E . No treatment is required
D . Pleural effusion
E . Pneumothorax * A 15 years old girl presents with history of fever ,
bleeding from gums and pallor for last 15 days . Her
Which of the following is least likely to cause peripheral blood films shows pancytopenia . The most
symmetrical weakness of the lower limbs ? important investigation is ?
A . Acute transverse myelitis A . Bone marrow examination
B . Glioblastoma multiforme B . Coombs test
C . Guillain barre syndrome C . Reticulocytes count
D . Myasthenia gravis D . Serum folic acid
E . Muscular dystrophy E . Serum iron level

* Elderly came with sudden loss of vision in right eye * The most sensitive vaccine to heat is ?
with headache , investigation show high CRP and A . IPV
high ESR . What is the diagnosis ? B . BCG vaccine
A . Temporal arteritis . C . Hepatitis B
B . SAH . D . OPV
C . Migraine . E . Diphtheria
D . Herpes ophthalmicus .
E . Tension headache . * In a patient with history of shortness of breath ,
which of the following signs indicates left heart
* Acute renal failure may be distinguished from failure ?
chronic renal failure by which of the following ? A . Ascites .
A . An increased Na excretion . B . Basal crepitations .
B . Left ventricular hypertrophy on ECG . C . Dependent edema .
C . Hypophosphatemia . D . Engorged neck veins .
D . Renal size on ultrasound scan . E . Fourth heart sounds .
E . Hyperkalemia .
* In a patient with high grade fever , rigors and
* In a patient with haematemesis . The clinical feature Hepatomegaly . The most likely diagnosis is ?
which suggests that peptic ulcer is underlying cause is A . Carcinoma of liver
? B . Liver abscess
A . Ascites C . Malaria
B . Drowsiness D . Right heart failure
C . History of jaundice E . Typhoid fever
D . Splenomegaly
E . Tender epigastrium

‫ اسامه الخزاعله‬: ‫الدكتور‬ ‫اسئلــــــــة الباطنيــــــة‬


Page 2
* Which one of the following is a contraindication to * A 40 years old lady presents with history of severe
thrombolysis ? constant upper abdominal pain ssociated with
A . Pregnancy vomiting , temperature is normal , and there is marked
B . Age over 75 years tenderness in epigastrium . Most seful investigation
C . Background diabetic retinopathy for this patient is ?
D . Asthma A . Cardiac enzymes
E . The presence of atrial fibrillation B . ECG
C . Gastroscopy
* A young boy presents in outdoor with history of D . Liver function test
sudden painful swelling of joints after minor trauma . E . Serum amylase
The most likely diagnosis is ?
A . DIC * A 67 years old man presents with bilateral ankle
B . ITP edema and pitting . Which of the following is the most
C . Sickle cell disease common cause ?
D . Hemophilia A A . Amlodipine
E . Glanzmann thrombasthenia B . Bisoprolol
C . Ruptured of Baker cyst
* CRP may be raised in all of the following except D . DVT
A . DM E . Cellulitis
B . Malignancy
C . Acute bacterial infection * A 50 year old smoker presents with history of cough
D . Tuberculosis productive of mucoid sputum every winter for last
E . Autoimmune disease three years . The most likely diagnosis is
A . Bronchial asthma
* In a young boy with HTN , examination CVS B . Chronic bronchitis
reveals radio _ femoral delay . The most common C . Bronchiectasis
cause of HTN in this patient is ? D . Bronchogenic carcinoma
A . Coarctation of aorta E . Pulmonary tuberculosis
B . Renal artery stenosis
C . Dissection of aorta * In a patient with history of muscle cramps and
D . Conn syndrome carpopedal spasm . Which of the following serum
E . Diabetic nephropathy electrolyte level is most likely to be decreased ?
A . Calcium
* A patient with history of jaundice , pruritus and clay B . Chloride
coloured stool . Which of the following enzymes may C . Magnesium
be increased significantly ? D . Potassium
A . ALP E . Sodium
B . AST
C . ALT * The anti diabetic agent of choice for 50 years old
D . LDH obese lady with mild hyperglycemia is ?
A . Chlorpropamide
* A patient presents with history of intermittent fever , B . Metformin
abdominal pain and headache . Blood culture is C . Insulin
positive for salmonella typhi . The antibiotic of choice D . Glibenclamide
for this patient ? E . Repaglinide
A . Ciprofloxacin
B . Vancomycin * A patient of chronic diarrhea is having angular
C . Metronidazole stomatitis and glossitis . The most likely cause of
D . Tetracycline these signs is deficiency of ?
E . Gentamicin A . Folic acid
B . Iron
* A 50 years old man is admitted in emergency ward C . Proteins
with MI . Any of the following medications are used D . Pyridoxine
initially ? E . Thiamine
A . Aspirin
B . Warfarin
C . Heparin
D . Streptokinase
E . Clopidogrel

‫ اسامه الخزاعله‬: ‫الدكتور‬ ‫اسئلــــــــة الباطنيــــــة‬


Page 3
* A 40 years old woman gives history of fever for last * An old patient presented in emergency ward with
three weeks accompanied by dry cough , night sweats history of weakness of right side of body of rapid
and weight loss . Abdominal examination reveals onest . The most helpful first line management of this
hepatosplenomegaly , CXR shows symmetrically patient is ?
distributed line nodules . A . Cerebral angiography
The most likely diagnosis is ? B . Cerebrospinal fluid examination
A . Pneumonia C . Computerised tomography scan brain
B . Malaria D . Fasting lipid profile
C . Typhoid E . Nerve conduction study
D . Chronic liver disease
E . Miliary tuberculosis * A 50 years old man presents with Dysphagia .
Which of the following characteristics suggests a
* A 40 years old diabetic man presents with history of benign strictures of esophagus ?
sudden onest of pain in right loin which radiates A . Anemia
towards the iliac fossa associated with vomiting .On B . Weight loss
examination tenderness is present in right loin . The C . Cervical lymphadenopathy
most likely diagnosis is ? D . Hoarseness of voice
A . Appendicitis E . Dysphagia worse for solids
B . Cholycystitis
C . Diverticulitis * Which of the following is a cause of central
D . Perforated peptic ulcer cyanosis ?
E . Pyelonephritis A . Exposure to cold
B . Heart failure
* A 20 year old Girl is taking anti tuberculosis C . Raynauds phenomenon
treatment , she presents in eye outdoor with visual D . Right to left cardiac shunts
complaints . The most likely cause of her symptoms is E . Shock
side effects of ?
A . Isoniazid * A young girl complains of nocturnal cough and
B . Pyrazinamide shortness of breath which disturbs her sleep . It was
C . Rifampicin diagnosed as bronchial asthma . Investigation to
D . Ethambutol confirm this diagnosis is ?
E . Streptomycin A . CXR
B . Eosinophil count
* A 40 years old man gives history of high grade fever C . Lung function test
for last one week with cough productive rusty sputum D . Serum IgG level
, breathing on right lower chest , CXR shows E . Sputum examination
consolidation . The most likely causative organism is
? * Which of the following does not form part of the
A . Anaerobic bacteria initial approach to investigating a fever of unknown
B . Gram negative bacilli origin ?
C . Streptococcus pneumonia A . Early CT scan of abdomen and chest
D . Staphylococcus aureus B . Serial blood cultures off antibiotics
E . Mycobacterium tuberculosi C . Careful history and examination
D . Autoantibody screen
* In a patient with microscopic Hematuria , which of E . Trial of antibiotics
the following favours a glomerular source ?
A . Red cell casts and proteinuria . * An epileptic girl is found to have gum hypertrophy .
B . Blood at the beginning of the urinary stream . Anti epileptic drug which she is most likely taking is ?
C . Blood at the end of the urinary stream . A . Carbamazepine
D . Suprapupic pain . B . Gabapentin
E . Abnormal urine cytology . C . Lamotrigine
D . Phenytoin
* An important physical sign of portal hypertension in E . Sodium valproate
a patient of cirrhosis of liver is ?
A . Gynecomastia
B . Hepatomegaly
C . Palmer Erythema
D . Spider angioma * A patient presents with tachycardia , wild swings in
E . Splenomegaly blood pressure , headache , diaphoresis , altered

‫ اسامه الخزاعله‬: ‫الدكتور‬ ‫اسئلــــــــة الباطنيــــــة‬


Page 4
mental status , and a sense of panic The most likely C . Venous thrombosis .
diagnosis is ? D . Hypocalcemia .
A . Hyperthyroidism . E . Pneumococcal infection .
B . Generalized panic disorder .
C . Angina pectoris . * Which of the following is a recognized feature of
D . Panic attack . massive pulmonary embolism ?
E . Pheochromocytoma . A . Reduced plasma lactate levels .
B . Increase in serum troponin levels .
* A 16 year old Girl presents with chronic diarrhea , C . An arterial PH less than 7.2 .
which of the following features suggest that she has D . Blood gases show increased PCO2 on air .
irritable bowel syndrome ? E . Normal D _ dimer levels .
A . Abdominal pain relieved by defecation .
B . Anemia . * A patient with embolic stroke is taking warfarin .
C . Weight loss . The most useful laboratory test to monitoring effect of
D . Nocturnal symptoms . warfarin ?
E . Blood in stools . A . PT
B . PTT
* The most common risk factor for COPD is ? C . Bleeding time
A . Air pollution D . Hematocrit
B . Infection E . Platelet count
C . Coal mining
D . Low socioeconomic status * A 68 year old patient with COPD was admitted with
E . Tobacco smoke a febrile exacerbation of his symptoms . Laboratory
showed arterial PH 7.21 , PCO2 60 mmhg , HCO3 24
* Which one of the following statements about meq/L . Which is the underlying acid base disorder ?
paracetamol poisoning is not true ? A . Respiratory acidosis .
A . 15 tablets may be a fatal overdose . B . Respiratory acidosis with metabolic Alkalosis .
B . Oral methionine may be useful in treatment . C . Respiratory acidosis with metabolic acidosis .
C . Paracetamol levels should be checked at 4 hours D . Metabolic acidosis with respiratory alkalosis .
post ingestion . E . Metabolic acidosis
D . If liver function tests are normal at 4 hours post
ingestion , the liver has not been damaged . * 35 years old man is admitted to the hospital and is
E . Activated charcoal should be given with undergo brain surgery . The patient is very anxious
laxatives . and worried of the upcoming surgery . He begins to
hyperventilate and becomes dizzy and unconscious ,
* A 35 years old man presents in a clinic with history the ABG reveal PH 7.61 , PCO2 22 mmhg , HCO3 24
of chronic productive cough that is worse in the meq/L . Most likely acid base disorder is ?
morning and that occurs because changing the A . Metabolic acidosis uncompensated .
position of patient . Sputum is copious and yellow . B . Respiratory Alkalosis compensated .
The most likely diagnosis in this patient is ? C . Respiratory Alkalosis uncompensated .
A . Bronchial asthma D . Metabolic Alkalosis compensated .
B . Bronchiectasis E . Metabolic Alkalosis uncompensated
C . Bronchogenic carcinoma
D . Pulmonary tuberculosis * Most common cause of Wallenberg syndrome is
E . Chronic bronchitis occlusion of ?
A . Basilar artery .
* A 50 year old man presents in emergency ward with B . Anterior cerebral artery .
central chest pain . On examination his blood pressure C . Middle cerebral artery .
90/60 and pulse 106 , and he is pale and sweating D . Posterior inferior cerebral artery .
profusely . The most likely diagnosis is ? E . Posterior cerebral artery
A . Pericarditis
B . MI
C . Esophagitis * A 5 years old boy gives history of swelling of body
D . Pleural effusion starting from face and more on getting up in the
E . Pneumothorax morning . On examination pallor is absent jugular
* The following are complication of Nephrotic venous pressure is not raised . Signs of Ascites and
syndrome with the exception of ? bilateral pleural effusion are found , the first line of
A . Acute renal failure . investigation is ?
B . Accelerated HTN . A . Blood urea level .

‫ اسامه الخزاعله‬: ‫الدكتور‬ ‫اسئلــــــــة الباطنيــــــة‬


Page 5
B . CXR . E . Left sided heart failure
C . Echocardiography .
D . Liver function test . * In patient with microscopic Hematuria , which of
E . Urine for albumin . the following favours a glomerular source ?
A . Red cell cast and protein in urine
* An 70 year old man presents with palpitations . He B . Blood at the beginning of voiding
has history of MI in 1995 and was placed on C . Blood at the end of voiding
nitroglycerin , Benzaepril , aspirin and verapamil . In D . Suprapubic pain
2000 he was placed on digoxin after a diagnosis of E . WBC in urine
heart failure was made . Three months ago he was
placed on Nebivolol and lasix ( Furosemide ) . He * A 25 year old man present to a rheumatologist with
presents with evidence of premature ventricular complaints of joint pain involving the large joints of
contractions , ST depression , and T waves inversion the legs . On questioning the patient indicates that
on the ECG , there is no chest pain . In mean time exacerbations in the joint pain are frequently
what should you do ? accompanied by diarrhea . Which of the following
A . Stop Benzaepril and digoxin . most likely diagnosis ?
B . Stop verapamil and lasix . A . Amebic colitis
C . Stop Nebivolol and lasix . B . Chronic appendicitis
D . Stop lasix and digoxin . C . Diverticulosis
E . Stop verapamil and aspirin . D . Pseudomembranous colitis
E . Ulcerative colitis
* A 52 year old man comes to the emergency
department because he has had vomiting, nausea, and * A 38 year old man has pulmonary tuberculosis ,
abdominal pain for the past 12 hours . He says he treatment with isoniazid , rifampin , ethambutol and
attempted suicide . 3 days ago by taking everything in pyrazinamide is started, which of the following should
the medicine cabinet . He was stuporous for be added to the medication regimen to prevent
approximately 12 hours after the overdose . But felt neurologic toxicity in this patient ?
better the following day . At this time , he has A . Vitamin B9
jaundice and pain in the RUQ . Which of the B . Vitamin B3
following drugs is most likely to have caused the pain C . Vitamin B12
, vomiting and jaundice ? D . Vitamin B6
A . Acetaminophen E . Vitamin C
B . Aspirin
C . Cimetidine * Major criteria of rheumatic fever are all except ?
D . Diphenhydramine A . Carditis
E . Triazolam B .subcutaneous nodules
C . Arthralgia
* A patient present with weakness , nausea , vomiting D . Erythema marginatum
, weight loss and new skin pigmentation Labs show E . Sydenham Chorea
Hyponatremia and Hyperkalemia , most likely
diagnosis is ?
A . Tinea versicolor
B . SIADH
C . Addison disease
D . Leprosy
E . Phenylketonuria
* A 30 year old female is started on Carbimazole 20
mg following a diagnosis of Graves disease . What is
the best biochemical marker to assess her response to
* A 70 year old man who is presented with 2 weeks treatment ?
history of abdominal pain that has worsened over the A . Total T4
last few days , on examination the patient is jaundice B . TSH
with tenderness over the epigastric area , with smooth C . Free T4
hepatomegaly and shifting dullness , which of the D . ESR
following is the cause of hepatomegaly ? E . Free T3
A . IDA
B . Budd Chiari syndrome * A 75 years old woman has increasing shortness of
C . Ulcerative colitis breath on exertion . Findings on physical examination
D . Crohn disease are unremarkable . X_Ray of the chest show no

‫ اسامه الخزاعله‬: ‫الدكتور‬ ‫اسئلــــــــة الباطنيــــــة‬


Page 6
abnormalities of the hear or lungs Labs show HCT E . Cataract
28% , Hb 9 g/dl , MCV 70 .
Which of the following is most likely diagnosis ? * All of the following can cause acute renal failure
A . Acquired Hemolytic anemia except ?
B . Chronic blood loss A . Rhabdomyolysis
C . Folic acid deficiency B . NSAIDs
D . B _ thalassemia minor C . Ethylene glycol
E . Pernicious anemia D . Penicillin
E . Iron
* What is the m.common cause of bacterial UTI ?
A . Staphylococcus * Which of the following is the most common cause
B . Neisseria of Duodenal ulcer ?
C . E coli A . NSAIDs
D . Streptococcus B . H pylori
E . Pseudomonas C . Alcohol abuse
D . Chronic corticosteroids
* Which of the following is an appropriate initial E . Zollinger Ellison syndrome
treatment for pulmonary tuberculosis ?
A . INH * All of the following forms of vaccine except ?
B . INH and rifampin A . Killed vaccine
C . INH , rifampin and ethambutol B . Attenuated vaccine
D . INH, rifampin , ethambutol and pyrazinamide C . Toxoid
E . INH , rifampin , ethambutol and streptomycin D . Serum
E . Subunit
* The treatment of choice for recurrent transient
ischemic attack in patient on aspirin with new onest * A 19 year old female presents with sudden sever
atrial fibrillation is ? right sided chest pain that develops shortly after she
A . Anticoagulation had been placing heavy boxes on shelves in her
B . Carotid endarterectomy garage physical examination reveals an a febrile
C . Clopidogrel female in mild respiratory distress . Breath sounds are
D . Corticosteroids markedly decreased on the right , and the right lung is
E . Carotid stent hyperresonant to percussion . Which of the following
is most likely present in this individual ?
* Chronic renal dialysis patient is brought to the A . Pneumoconiosis
emergency department incardiac arrest , the most B . Pneumocystis infection
likely cause is ? C . Bacterial pneumonia
A . Pericardial effusion D . Viral pneumonia
B . Hyperkalemia E . Pneumothorax
C . Hypocalcemia
D . Malignant HTN
E . Post dialysis hypotension

* Tertiary prevention consist of ?


* All of the following are contraindications to passing A . Crisis preparation
a nasogastric tube except ? B . Elimination of specific disease
A . Hx of esophageal varices C . Early diagnosis
B . Basal skull fracture D . Rehabilitation after disability
C . Suspected perforation of the esophagus E . Environmental sanitation
D . Complete intestinal obstruction
E . Presence of esophageal foreign body * Which of the following statement about Polyvalent
anti_snake venom serum is true ?
* All of the following are side effects of steroid except A . It is to be given subcutaneously
? B . It is an example of passive immunization
A . Elevated blood pressure C . It is usually obtained from human's donor's
B . Acne D . Should be stored at 0 Sep Celsius
C . Skin bruises E . It gives life long immunity
D . Hypoglycemia

‫ اسامه الخزاعله‬: ‫الدكتور‬ ‫اسئلــــــــة الباطنيــــــة‬


Page 7
* All are true about CPR quality in advanced cardiac * NT_PRO BNP elevation is caused by all of the
life support except ? following except ?
A . Push hard less than 2 inches and allow A . MI
complete chest recoil . B . CHF
B . Push fast more than 100 and less than 120 C . Atrial fibrillation
compressions per minute . D . PE
C . Minimize interruptions in compressions and give E . COPD exacerbation
shock as soon as possible .
D . Compressions to ventilation ratio 30 : 2 . * All are true about Hyperosmolar hyperglycemic
E . Target of CPR quality are ETco2 > 10 mmhg and state except ?
diastolic BP > 20 mmhg . A . Partial or relative insulin deficiency .
B . Characterized by hyperglycemia ,
* All of the following are common causes of chest hyperosmolarity , dehydration and ketosis .
pain except ? C . Occurs in DM type two .
A . Angina D . Lower insulin requirement compared to DKA .
B . Pericarditis E . Overall mortality higher compared to DKA .
C . PE
D . Tension pneumothorax * All are causes of Acute bloody diarrhea except
E . Esophageal reflux A . Shigella
B . Campylobacter
* All of the following are true matching about drugs C . Bacillus cereus
and side effects except ? D . Yersinia
A . Sulfa drugs _ Steven Johnson syndrome . E . Entamoeba histolytica
B . Aminoglycosides _ nephrotoxicity .
C . Acetaminophen _ GI upset and bleeding . * All of the following are indications for Enalapril
D . Opiates _ constipation . administration except ?
E . Atenolol _ Bradycardia. A . HTN
B . MI
* All of the following are causes of DM except ? C . CHF
A . Thyroid hormone D . DM
B . Thiazides E . Aortic stenosis
C . Pancreatic cancer
D . Addison disease * All are risk factors for Barrett esophagus except
E . Hemochromatosis A . Old age
B . Female
* All of the following are differential diagnosis of ST C . Smokers
segment elevation except ? D . Obesity
A . Acute Pericarditis E . Hiatus hernia
B . Early repolarization
C . Hyperthermia
D . Prinzmetal angina
E . Left ventricular aneurysm

* All of the following are rapid or short acting * All are true about Crohn disease except ?
prandial bolus insulin except ? A . Usually affect terminal ileum .
A . Insulin glargine B . Fever is very common .
B . Insulin regular C . Palpable mass is rare compared to ulcerative
C . Insulin lispro colitis .
D . Insulin glulisine D . Diarrhea less prevalent than ulcerative colitis
E . Insulin aspart E . Cobblestone mucosal appearance .

* Adverse consequences of obesity include all of the * All of the following are true about irritable bowel
following except ? syndrome except ?
A . Coronary artery disease A . Onest of symptoms usually in young adults .
B . Hypertension pulmonary B . Signs and symptoms relived with defecation ,
C . DM type one increased by stress .
D . Osteoarthritis C . Bloody diarrhea .
E . Colorectal cancer D . Inflammatory markers are negative .
E . 80% improve over time .

‫ اسامه الخزاعله‬: ‫الدكتور‬ ‫اسئلــــــــة الباطنيــــــة‬


Page 8
* All of the following are matching about hepatitis B * All of the following are true matching about RBC
serology except ? inclusions except ?
A . Acute infection _ HBsAg , HBeAg , anti HBc A . Erythroblasts _ Hemolytic anemia .
IgG are positive . B . Heinz bodies _ IDA
B . Chronic inactive infection _ HBsAg , HBeAg , C . Howell jolly bodies _ post splenectomy
anti HBc IgG are positive . D . Basophilic stippling _ lead poisoning .
C . Chronic active infection _ HBsAg , anti HBe , anti E . Sideroblasts _ hypothyroidism
HBc IgG are positive .
D . Resolved infection _ anti HBs , anti HBe , anti * Westermark sign , Hamptons hump , enlarged right
HBc IgG are positive . ventricle and right atrium and atelectasis on CXR is
E . Immunized person _anti HBs positive . characteristic for ?
A . Pulmonary edema
* Auer rod white blood cell on blood film is B . Pulmonary embolism
characteristic for ? C . Sarcoidosis
A . AML D . COPD
B . CML E . Pulmonary fibrosis
C . ALL
D . CLL * All of the following are causes of Hyponatremia
E . Hodgkin lymphoma except ?
A . Nephrotic syndrome
* Mode of transmission for all of the following by B . Hyperthyroidism
food_water borne except ? C . SIADH
A . MRSA D . Furosemide
B . Cholera E . CHF
C . Salmonella
D . HAV * Signs and symptoms of third cranial nerve palsy
E . HEV include all of the
following except ?
* Red currant jelly sputum is characteristic for ? A . Absent of light reflexes
A . Streptococcal pneumonia B . Ptosis
B . Staphylococcal pneumonia C . Eye deviated downward
C . Klebsiella pneumonia D . Eye deviated outward
D . Mycoplasma pneumonia E . Mydriasis
E . Legionella pneumonia
* Paresthesia , sensory ataxia , myopathy , subacute
* All of the following are causes of Reticulocytosis combined degeneration of spinal cord , optic
except ? neuropathy are characteristic of ?
A . SCA A . Vitamin B12 deficiency
B . Chronic blood loss B . Vitamin B6 deficiency
C . HUS C . Vitamin B1 deficiency
D . DIC D . Folate deficiency
E . Malaria E . Niacin deficiency
* All of the following are causes of cavitating lung * All are causes of Hypokalemia except ?
nodule on CXR except ? A . Bartter syndrome
A . Wegener disease B . Angelman syndrome
B . Allergic alveolitis C . Hypomagnesemia
C . Sarcoidosis D . Conn syndrome
D . Tuberculosis E . Cushing syndrome
E . Metastasis
* One is true about Petit mal seizures ?
* All of the following are true matching about the A . Muscle rigidity
interpretation of urinary casts except ? B . Tongue biting
A . Fatty casts _ nephritic syndrome . C . Arrest of activity for 5 _ 10 seconds .
B . Pigmented epithelial granular cast _ ATN . D . Flaccid limbs
C . White blood cell casts _ interstitial nephritis . E . Loss of muscle tone
D . Red blood cell casts _ glomerulonephritis .
E . Hyaline casts _ physiological during exercise or * The goal of tertiary level of prevention is ?
fever A . Protect health and lifestyle

‫ اسامه الخزاعله‬: ‫الدكتور‬ ‫اسئلــــــــة الباطنيــــــة‬


Page 9
B . Prevention of disease Onest A .Thyrotoxicosis
C . Early detection of disease B .Multiple myeloma
D . Treat and rehabilitate of disease to prevent C . Lung cancer
progression , and disability . D . Furosemide
E . Stop of smoking . E . Hyperparathyroidism

* All of the following are causes of Hyperglycemia * 55 years old male patient with with back pain , spine
except ? X_Ray shows fracture in body of lumbar vertebrae .
A .DM Lab findings : K 4.1 , Na 140 , Cl 101 , Ca 13 , Hb 8.5
B .Pheochromocytoma , WBC 12.3 , platelets 150*10`9 , total protein 105
C .Thyrotoxicosis g/dl . Which of the following is the most likely
D . DI diagnosis ?
E . Glucagonoma A .Sarcoidosis
B . Osteosarcoma
* All of the following are causes of hypoglycemia C . Multiple myeloma
except ? D . Age related changes
A .Glucagonoma E . Paget disease of the bone
B .Postprandial
C .Addison disease * The main pathophysiology of Myasthenia gravis is
D . Glibenclamide ?
E. Liver cirrhosis A . Anti calcium channel bodies
B . Low level of acetylcholine
* All of the following are causes of Hyponatremia C . Hyposensitivity to acetylcholine
except ? D . Acetylcholine receptor antibodies
A . SIADH E . Neuromuscular degeneration changes
B . Furosemide
C . DKA * All are causes of Polyuria except ?
D . DI A . DM
E . Addison disease B . DI
C . Renal failure
* About SIADH , all are true except ? D . Hypercalcemia
A . Euvolemic state E . SIADH
B . High urine specific gravity
C . High urine sodium * All are true about resistant HTN except ?
D . Hypernatremia A . It is diuretics related condition
E . Low serum osmolality B . Pheochromocytoma is underlying cause .
C . To put this diagnosis , patient must be on 3 drugs ,
* Most c. cause of hypothyroidism worldwide ? one of them Thiazide .
A . Hashimoto thyroiditis D . To put this diagnosis , patient must be take the
B . Gravis disease optimal dose for HTN drugs .
C . Iodine deficiency E . Diet problems is underlying cause .
D . Subacute thyroiditis
E . Post_thyroidectomy
* The best laboratory test for follow up patient with * All are causes of Thrombocytosis except ?
Hashimoto thyroiditis is ? A . IDA
A . T3 B . Polycythemia rubra vera
B . T4 C . Hypersplenism
C . TSH D . After Splenectomy
D . TRH E . Collagen disorders
E . ESR
* All of the following are causes of Hypokalemia
* All are true about hyperthyroidism except ? except ?
A .Increased appetite A . Cushing syndrome
B . Heart rate 50 BPM B . Conn syndrome
C . Hypomenorrhea C .Thiazide
D . TSH is very low D .Pyloric stenosis
E . Atrial fibrillation on ECG E . Addison disease

* All are causes of hypercalcemia except ?

‫ اسامه الخزاعله‬: ‫الدكتور‬ ‫اسئلــــــــة الباطنيــــــة‬


Page 10
* All of the following are causes of Hyperkalemia A . This disease is preventable and treatable .
except ? B . This disease is systemic
A . Nelson syndrome C . This disease mainly affect the Lower lobes of
B . Rhabdomyolsis the lung
C . Digoxin toxicity D . BCG vaccine is live attenuated vaccine for TB
D . Enalapril E . Most patients TB infections are due to secondary
E . Conn syndrome infection

* One of the following statements is wrong ? * Obesity associated with all of the following except ?
A . Patient with Pheochromocytoma must be A . Psychological disorders
treated by B_Blockers B . GIT disturbances
B . Pheochromocytoma is one of the causes of C . Colorectal cancer
resistant HTN . D . Osteoporosis
C . Most common cause of secondary HTN is renal E . Osteoarthritis
disease .
D . Conn syndrome is primary hyperaldosteronism . * All are causes of Leucocytosis except ?
E . Most common cause of Thyrotoxicosis is Graves A . Dermatomyositis
disease . B . Polymyositis
C . Fibromyalgia
* Amlodipine is ? D . Septic arthritis
A . Aldosterone antagonist . E . Polyarteritis nodosa
B . B blockers .
C . Muscle relaxants . * All are causes of microcytosis except ?
D .Dihydropyridine CCB A . Pernicious anemia
E . Non Dihydropyridine CCB . B . Sideroblastic anemia
C . B thalassemia
* 50 years old male patient is presented with fasting D . Lead toxicity
blood sugar 139 mg/dl , which of the following is E . Anemia of chronic disease
most appropriate next step ?
A . GAD study . * 22 years old patient with celiac disease , admitted
B . Postprandial glucose tolerance test . with chief complain of diarrhea for 2 weeks , the most
C . Start treatment right now as DM . likely cause is ?
D . Another reading of fasting blood sugar A . GIT infection
E . Diet therapy . B . IBS
C . IBD
* All are true about DKA except ? D . Carcinoid syndrome
A . PH of the serum < 7.35 E . No adherent to gluten free diet
B . Hyponatremia
C . Ketones bodies in serum
D . Normal anion gap
E . Complicated by brain edema * 25 years old male patient present with multiple
psychological problems , which of the following
* The major deference between primary and disturbances is the most likely to see in this patient ?
secondary adrenal insufficiency is ? A . Bloody diarrhea
A . Na and k in serum B . GIT ulcers
B . Blood pressure C . Occult blood in stool
C . Weight status D . Alternating diarrhea and constipation
D . Glucose in serum E . IDA
E . Skin pigmentation
* Most common cause of upper GI bleeding is ?
* Criteria of disease activity for rheumatoid arthritis A . Mallory Weiss syndrome
include all of the following except ? B . PUD
A . Rheumatoid factor C . Varicose veins of esophagus
B . ESR D . Esophagitis
C . CRP E . Gastric angiodysplasia
D . Thrombocytosis
E . Anti_CCP * Best tool for diagnosis of infective endocarditis
A . ECG
* All of the following about TB are true except ? B . CT angiography

‫ اسامه الخزاعله‬: ‫الدكتور‬ ‫اسئلــــــــة الباطنيــــــة‬


Page 11
C . Mediastinoscopy C . Aneurysm rupture
D . MRI D . Trauma
E . Two dimensional echo E . Vasculitis

* All are causes of chronic liver disease except ? * All are causes of increased CVP except ?
A . Hepatitis B A . Fluid overload
B . Hemochromatosis B . Right sided heart failure
C . Alpha one antitrypsin deficiency C . Pulmonary HTN
D . Hepatitis A D . Anaphylactic shock
E . Cystic fibrosis E . Pulmonary stenosis

* All are complications of cystic fibrosis except ? * All are used in treatment of Anaphylactic shock
A . Pancreatic insufficiency except ?
B . Hypoglycemia A . Adrenaline
C . Recurrent lung disease B . IV fluid
D . Recurrent sinusitis C . 100 % oxygen
E . Anal prolapse D . Hydrocortisone
E . Propranolol
* All are causes of Bradycardia except ?
A . Metoprolol * The major deference between vitamin B12 and
B . Digoxin folate deficiency anemia is ?
C . Thyroxine A . MCV value
D . Athletic person B . HCT value
E . Neostagmine C . Blood film findings
D . Foot Paresthesia
* Risk of transmission of HIV to medical person's E . MCHC value
from needle stick is ?
A . 0.3% * All are true about Mycoplasma pneumonia except ?
B . 0.03% A . Patient is looking well .
C . 3% B . Associated with Hemolytic anemia .
D . 30% C . Common in younger age group .
E . 10% D . Good response for Macrolides antibiotics .
E . Associated with septic shock
* About HIV , all are true except ?
A . AIDS is the last stage of HIV infection .
B. HIV infection is not preventable by vaccination
C . Heterosexual type of transmission is the most
common * Highly severe pneumonia depends on all of the
D . It is transmitted by blood transfusion . following criteria except ?
* Paracetamol antidote is ? A . BUN > 7 mmol/l
A . Protamine sulfate B . Confusion
B . N_acetylcysteine C . Age 20_30 years old
C . Flumazenil D . R.R > 30/min
D . Bicarbonate E . Systolic blood pressure < 90 mmHg
E . Vitamin K
* All of the following are true about Nephrotic
* About pulmonary embolism , all are true except syndrome except ?
A . Past history of DVT support the diagnosis . A . HTN
B . Heparin is used for treatment B . Proteinuria > 3.5 gram/day
C . Chest X_Ray usually normal without significant C . Edema
changes D . Hypercholesterolemia
D . It is a block in one or more of the pulmonary E . Hypoalbuminemia
veins
E . Classical ECG finding is S1Q3T3 * The most important finding in Glomerulonephritis is
?
* Most important risk factors for stroke development A . WBCs casts in urine
is ? B . Hyaline casts in urine
A .High blood pressure C . RBCs casts in urine
B . Low blood pressure D . Proteinuria

‫ اسامه الخزاعله‬: ‫الدكتور‬ ‫اسئلــــــــة الباطنيــــــة‬


Page 12
E . Bacteriuria
* All of the following are increases risk of DVT
* Best drug that is used for prophylaxis and treatment except ?
of diabetic nephropathy is ? A . Malignancy
A . ARBs B . Estrogen therapy
B . B blockers C . Old age
C . CCB D . Protein C excess
D . Furosemide E . Protein S deficiency
E . Nitrofurantoin
* In Hyperventilation syndrome , one is true ?
* Best way of the following to prevent diabetic A . Increasing PH of the serum
retinopathy is ? B . Caused by severe attack of bronchial asthma
A . Optimal dose of insulin C . Characterized by hypercapnia
B . Optimal dose of metformin D . Caused by poliomyelitis
C . Annual visit to physician E . No relation to acute respiratory distress syndrome
D . ACE_i administration
E . Good control of sugar * 65 years old male patient , present to emergency
room with chief complain of right sided weakness and
* The most early and best laboratory finding for speech inability , one physical examination, right side
diagnosis of diabetic nephropathy is ? weakness 1/5 , speech exam , patient obey command
A . Urine analysis positive for Bacteria but can't speak any words , this type of dysphasias is
B . Kidney biopsy most likely ?
C . RBCs casts in urine A . Receptive aphasia
D . 24 hour protein /creatinine ratio in the urine B . Conduction aphasia
E . Kidney ultrasound C . Expressive aphasia
D . Nominal aphasia
* All of the following are causes of increasing E . Dysarthria
Triglycerides in serum except ?
A . Nephrotic syndrome All are components of Horner syndrome except ?
B . Ibuprofen A . Proptosis
C . DM B . Myosis
D . Alcohol C . Anhidrosis
E . Lipoprotein lipase deficiency D . Enophthalmos
* All of the following are decrease potassium in
Hyperkalemic patient except ?
A . Insulin and glucose
* All of the following represents targets for systemic B . Ventolin nebulizer
HTN except ? C . Furosemide
A . Retinopathy D . Calcium gluconate
B . Nephropathy E . Kayexalate resin
C . Heart failure
D . Subdural hemorrhage * All of the following are causes of huge massive
E . Peripheral vascular disease Splenomegaly except ?
A . Hemolytic anemia
* All of the following are causes of exudative pleural B . Myelofibrosis
effusion except ? C . Malaria
A . PE D . Leishmaniasis
B . Hypoalbuminemia E . Tropical Splenomegaly
C . Lymphoma
D . Mesothelioma * All of the following are risk factors for hepatic
E . SLE encephalopathy except ?
A . GIT bleeding
* All of the following are true about COPD , except ? B . Benzodiazepine
A . FEV1< 80% of predicted C . Rifampicine
B . Complicated by right heart failure D . Diuretics
C . High F:O2 is needed E . Neomycin
D . Stop of smoking is beneficial in stop progression
of the disease * All are true about acute renal failure except ?
E . In ABG PaO2 is low , PaCO2 is high . A . Pre_renal causes are the m. common causes

‫ اسامه الخزاعله‬: ‫الدكتور‬ ‫اسئلــــــــة الباطنيــــــة‬


Page 13
B . ATN is the most common renal cause It teratogenic , so it can't be given in gestational
C . It is rarely caused by enlarged prostate diabetes .
D . Small kidneys and increased echogenicity on Most serious side effects is lactic acidosis , so it can't
ultrasound be given in renal failure .
E . Usually no history of Co_morbidity It inhibits gluconeogenesis in the liver .
No effect on pancreas .
* Long term steroid therapy can cause all of the
following except ? * Which of the following causes of Potts disease
A . Hypernatremia Primary TB
B . Hyperkalemia Skeletal TB
C . GIT ulcers TB meningitis
D . Suprarenal gland failure Miliary TB
E . Osteoporosis GIT TB

* About DKA , all are true except ? * Patient presents with COPD , Hepatomegaly,
A . Hyponatremia peripheral edema , finger clubbing . ECG : peaked P
B . Leucocytosis wave , most likely diagnosis ?
C . Serum PH decreased PE
D . PaCO2 decreased Cor pulmonale
E . Normal anion gap LVF
LAH
* In severe bronchial asthma , all of the following are Sleep apnea
true except ?
A . Cyanosis * All of the following are true about Sarcoidosis
B . Confusion except ?
C . Hyperresonant chest More in black race .
D . Exhaustion May cause complete heart block .
E . Bradycardia Involve mostly left hilar lymph nodes .
Lupus pernio and Erythema nodosum are seen .
Hepatosplenomegaly is common .

* The target HbA1c in diabetic patient is ?


A . < 6% * The criteria for diagnosis chronic bronchitis is ?
B . < 7% A . Cough productive for 2 months of two successive
C . < 8% years .
D . < 9% B . Cough productive for 2 months of three successive
E . < 5% years .
C . Cough productive for 3 months of two
* All are causes of Hypokalemia except ? successive years .
A . Spironolactone D . Cough productive for 3 months of three successive
B . Furosemide years .
C . Thiazide
D . Conn syndrome * Drug of choice to treat patient with HTN and
E . Cushing syndrome bronchial asthma is ?
ACE_i
* All are true about inflammatory diarrhea except B blockers
A .Stool PH is low with alkalosis . CCB
B .Stool usually watery . Statin
C.Consider IBD, c.Difficile infection .
D .Stool volume less than 500 ml . * All are true about Ankylosing spondylitis except
E .Continue during fasting, associated with anemia , A . More common in male .
urgency . B . Aortitis and AV block are manifestation .
C . Unilateral uveitis with painful red eye ,
* All are true about metformin except ? Photophobia .
Decreased insulin resistance , so it can be given in D . Obstructive lung disease is manifestation .
PCOS . E . Associated with HLA_B27 gen .

‫ اسامه الخزاعله‬: ‫الدكتور‬ ‫اسئلــــــــة الباطنيــــــة‬


Page 14
* All of the following are characteristic of IDA except Levothyroxine
? Levofloxacine
Anisocytosis
Spherocytosis * Most common site of tuberculosis in Jordan is ?
Poikilocytosis Skeletal
Hypochromia Meningitis
Microcytosis Lymphatic
Pulmonary
* Most common cause of travelers diarrhea is ? Miliary
E coli
Cholera * Most common antibiotics to cause anaphylaxis
Shigella Aminoglycosides
Brucella B_lactam
Staphylococcus aureus Cephalosporin
Glycopeptide
* One of the following seen in esophageal perforation Macrolides
?
Courvoisier sign * An injury of optic chiasm by pituitary adenoma will
Grey sign cause ?
Cullen sign Monocular visual loss
Boerhaave syndrome Bitemporal hemianopia
Achalasia Homonymous hemianopia
Inferior Homonymous quadratanopia
* All are signs and symptoms of hypothyroidism
except ? * Which of the following muscles is supplied by
Carpal tunnel syndrome Abducens nerve ?
Hyponatremia Superior rectus
Pericardial effusion Medial rectus
Hypolipidemia Lateral rectus
Normochromic Normocytic anemia Inferior rectus
Superior oblique

* Most specific test for diagnosis Pheochromocytoma


* Most common cause of gynecomastia is ? is ?
Liver failure CT scan
Estrogen secreting tumor Dexamethasone test
HCG secreting tumor 24 hour urine metanephrines
Idiopathic Plasma free metanephrines
Puberty
* The most appropriate drug to use in case of thyroid
* All are characteristic for MEN1 except ? storm is ?
Hypercalcemia Methimazole
Hypoglycemia Dexamethasone
PUD Propylthiouracil
Pheochromocytoma Supportive care
Glucagonoma Iodine

* Most effective drug increasing HDL is ? * All of the following are true about DKA except ?
Statins Hypovolemia
Fibrate Kassmaul breathing
Bile acid resins Coma
Niacin Hyperglycemia
Ezetimibe Low anion gap metabolic acidosis

* Red man syndrome is a side effect of ? * All are causes of Hypocalcemia except ?
Vancomycin Acute pancreatitis
Amoxicillin Hypermagnesemia
Metronidazole Pseudo_hypoparathyroidism

‫ اسامه الخزاعله‬: ‫الدكتور‬ ‫اسئلــــــــة الباطنيــــــة‬


Page 15
Hyperphosphatemia Cellulitis
Chronic renal failure Ruptured Baker cyst
Lymphedema
* First step in diagnosis Acromegaly is one of the Left heart failure
following test in serum ? DVT
IGF_1
IGF_2 * Which of the following is contraindicated in case of
G.H variant prenzmetal
G.H.R.H angina ?
Somatostatin ACE_i
ARBs
* Most common manifestation of Brucellosis ? B blockers
Bradycardia CCB
Abdominal pain GTN
Fever
Dry cough * Most common cause of secondary HTN is ?
Skin rash Chronic kidney disease
Cushing syndrome
* Most common cause of Cor pulmonale is ? Hyperthyroidism
Pulmonary embolism Obstructive sleep apnea
Obstructive sleep apnea Conn syndrome
COPD
Bronchial asthma * Most common cause of infective endocarditis ?
Tuberculosis Streptococcus group A
Streptococcus group B
* All of the following are symptoms of bronchial Streptococcus viridans
asthma except ? Staphylococcus viridans
Dry cough Staphylococcus group A
Chest tightness
Wheezing
Stridor
Shortness of breath

* All are true about Sarcoidosis except ? * Most common way to diagnose aortic stenosis
A . Restrictive lung disease . CT angiography
B . Hypocalcemia . MRA
C . Non caseating Granuloma on biopsy . Transesophageal echocardiography
D . Hepatosplenomegaly . Aortoangiography
E . Painless skin sores on face which called lupus
pernio . * All of the following are side effects of Thiazide
diuretic except ?
* All of the following are signs of pulmonary Hypokalemia
hypertension except ? Hypocalcemia
A . Right ventricular hypertrophy . Hypomagnesemia
B . Prominent A wave on JVP . Hyponatremia
C . Loud S1 Hyperglycemia
D . Hepatosplenomegaly and Ascites .
E . Systolic pulmonary pressure > 30 mmHg and * Drug of choice to treat Raynauds phenomenon
mean pulmonary pressure > 20 ACE_i
mmHg . B blockers
CCB
* LMWH works on ? Loop diuretics
Factor 2 ARBs
Factor 7
Factor 8 * All of the following are parts of atrial fibrillation
Factor 10 management except ?
Factor 12 Class 1A Quinidine
Class 1B lidocaine
* All are causes of lower limbs edema except ? Class 1C propafenone

‫ اسامه الخزاعله‬: ‫الدكتور‬ ‫اسئلــــــــة الباطنيــــــة‬


Page 16
Class 3 Amiodarone Neurology
DC shock Renal
Blood
* All of the following must be treated with DC shock
except ? * All are true about reactive arthritis Reiters syndrome
Unstable atrial fibrillation except ?
Pulsless ventricular tachycardia Keratoconjunctivitis sicca and uveitis .
Ventricular fibrillation Balanitis and oral ulcer .
Unstable fibrillation HLA_B27 is positive and RF is positive .
PVC GIT infections are the only cause .
Kerato_derma blenorrhagica
* Carotid thrills , arterial pulsation in retinal artery (
Becker sign ) , head nodding , water hummer pulse , * Most specific antibodies in SLE ?
bisfirenes pulse , pistol shot pulse are Anti ss _ A
seen ? Anti ss _ B
Mitral stenosis Anti _ RNP
Mitral regurgitation Anti Ds_DNA
Aortic stenosis ANA
Aortic regurgitation
Pulmonary stenosis * Which of the following large vessel pathology ?
A . HSP
* Most common valve disease causing pulmonary B . Takayasu arteritis
HTN ? C . Wegener Granulomatosis
Mitral stenosis D . Churg Strauss syndrome
Mitral regurgitation E . Kawasaki disease
Aortic stenosis
Aortic regurgitation * Most specific test for diffuse systemic sclerosis is ?
Mitral valve prolapse SCL_70
Anti_centromere antibodies
ANA
Anti RNP
Anti histone antibodies
* All are true about Behcets disease except ?
* All are risk factors for developing infective A . Most common manifestation is painful oral
endocarditis except ? apththous ulcer .
AIDS B . Erythema toxicoum
VSD C . DVT and PE
HOCM D . HLA_B51 gen is positive
ASD E . Mostly seen in Turkish people age 20_30 years old
Prosthetic valve .

* Most sensitive test to diagnose SLE is ? * Philadelphia chromosome is mostly seen in which of
RF the following ?
ESR A . Acute myeloblastic leukemia .
Ds_DNA B . Acute lymphoblastic leukemia .
ANA C . Chronic myeloblastic leukemia .
Anti_smith D . Hodgkin lymphoma
E . Non Hodgkin lymphoma
* Which of the following can be used for diagnosis of
Wegener Granulomatosis ? * Carbamezipine can be used in all of these cases
C_ANCA antibodies except ?
P_ANCA antibodies Partial seizure simple
Anti_RNP Generalised tonic clonic seizure
ANA antibodies Myoclonic or absence seizure
Anti histone antibodies Trigeminal neuralgia
Complex partial seizure
* Most common SLE manifestation is ?
Skin * An injury to the optic tract will cause ?
Joints A .Monocular visual loss

‫ اسامه الخزاعله‬: ‫الدكتور‬ ‫اسئلــــــــة الباطنيــــــة‬


Page 17
B .Bitemporal Hemianopia X Ray
C .Homonymous hemianopia MRI
D .Inferior Homonymous quadratanopia Angiogram
E.Superior Homonymous quadratanopia MRA

* Which of the following can be used to differentiate * All are true about Parkinson disease except ?
between Myasthenia gravis and Lambert Eaton A . Degeneration of extrapyramidial pathway .
Myasthenic syndrome ? B . Decreased dopamine level in brain .
X_Ray C . Triad Akinesia , tremor resting , rigidity .
CT scan D . Mean age of onest 60 years old .
MRI E . Pathology is found in substantia gelatinosa .
ECG
EMG * Most common cause of meningitis in epidemic area
is ?
* First line drug to be used in management a patient Neisseria meningitis
with migraine headache is ? Streptococcus pneumonia
Ergotamine Tuberculosis
Sumatriptan Hib
Propranolol Staphylococcus aureus
Verapamil
Lithium * Drug of choice for prevention of close contacts with
patient with
* Most common cause of epilepsy is ? meningococcal meningitis is ?
Stress Corticosteroids
Congenital Ceftriaxone
Trauma Rifampicine
Idiopathic Benzylpenicillin
Hypoxia Vancomycin

* All of the following are true about Brown Sequard


* The treatment of choice of Juvenile Myoclonic syndrome except ?
epilepsy ( Janz syndrome ) is ? A . Upper motor neuron weakness on the same side as
Carbamezipine the lesion .
Sodium Valproate B . Loss of position and vibration ipsilateral to the
Phenytoin lesion .
Corticosteroids C . Touch in intact .
Clonazepam D . Loss of pain and temperature ipsilateral to the
lesion .
* All are true about migraine headache except ?
A . More common in teenage men . * Uhthoff phenomenon is seen in ?
B . Stress , exercise , cheese , chocolate , menarche Parkinson disease
are triggering factors . Meningitis
C . Associated with aura . Multiple sclerosis
D . Unilateral associated with vomiting and Guillain barre syndrome
Photophobia . Duchenne muscular dystrophy
E . Maximum last for 3 days .
* Which of the following disease are associated with
* The treatment of choice for infantile spasms ( west both degeneration in
syndrome ) ( Hypsarrhythmia on ECG ) is ? upper motor neuron and lower motor neuron ?
Carbamezipine Parkinson disease
Sodium Valproate Multiple sclerosis
Phenytoin Alzheimer's disease
Corticosteroids Amyotrophic lateral sclerosis
Clonazepam Duchenne muscular dystrophy

* Best way to differentiate between cerebral infraction * ALL of the following are associated with SLE
and hemorrhage is ? except :
CT scan A) Treatment include methotrexate

‫ اسامه الخزاعله‬: ‫الدكتور‬ ‫اسئلــــــــة الباطنيــــــة‬


Page 18
B ) Cardiovascular disease is the most common cause A) DI
of death . B) Renal failure
C) erosive arthritis C) Insulin deficiency
D) Libman sacks endocarditis D) SIADH
E) Malar butterfly rash E) Hypercalcemia

* All of the following are causes hyperurecemia * All are side effects of digitalis toxicity except :
except : A) Nausea and vomiting
A) Hypothyroidism B) Yellow vision
B) Lesh Nyhan syndrome C) Short PR interval
C) Thiazide diuretic D) Gynecomastia
D) Hyperthyroidism E) Anorexia
E) INH
* All of the following are indication for liver biopsy
* Miosis associated with all of the following except : except :
A) Organophosphates poisoning A) Hepatic tumor
B) Morphine B) Wilson's disease
C) Oculomotor nerve palsy C) Prolonged PT
D) Horner syndrome D) Fever unknown origin
E) Cluster headache E) Autoimmune hepatitis

* All of the following are associated with diarrhea * Which of the following is CCB :
except : A) Furosemide
A) Hyperparathyroidism B) Diltiazem
B) Celiac disease C) Digoxin
C) Inflammatory bowel disease D) Simvastatin
D) Gastrinoma E) Ramiprile
E) Cryptosporidium * Which of the following valvular heart disease
increase risk of angina :
* All are true about multiple sclerosis except : A) Aortic regurgitation
A) Occur in elderly B) Mitral regurgitation
B) Remission and relapse C) Aortic stenosis
C) Unilateral optic neuritis D) Mitral stenosis
D) Antibodies to oligodendrocyte glycoprotein E) Pulmonary stenosis
E) Lhermitts sign and uhthoffs sign are positive
* All of the following are causes of chest pain except :
* Most common cause of early death postoperative MI A) Dissecting aortic aneurysm
: B) PE
A) Ventricular extrasystole C) Pericarditis
B) Myocardial rupture D) Cardiomyopathy
C) Myocardial aneurysm E) Tietzes syndrome
D) Ventricular arrhythmia
E) Cardiac tamponade * All of the following associated with aortic stenosis
except :
* First line in management of ventricular fibrillation : A) Sudden death
A) Adrenaline B) Wide pulse pressure
B) Atropine C) Slow rising pulse
C) Amiodarone D) P mitrale and LAD on ECG
D) DC shock E) Syncope
E) Lidocaine
* All of the following are causes of huge
* All of the following feco_oral transmission except : Splenomegaly except :
A) HEV A) CML
B) HDV B) Kala_azar
C) Poliomyelitis C) Malaria
D) Gardiasis D) Sarcoidosis
E) Cryptosporidium E) Infectious mononucleosis

* All of the following causes Polyuria except :

‫ اسامه الخزاعله‬: ‫الدكتور‬ ‫اسئلــــــــة الباطنيــــــة‬


Page 19
* All of the following are causes of Extrasystole * One of the following associated with Addison
(PVCs) except : disease :
A) Digoxin toxicity A) Hypokalemia
B) MI B) Hyponatremia
C) Lidocaine C) Hypoglycemia
D) Heart failure D) HTN
E) HTN E) Weight gain

* All of the following are associated with * Gold standard for diagnosis of erosive gastritis :
Parkinsonism except : A) Gastric PH_metry
A) Bradykinesia B) Gastric biopsy
B) Mask like face C) Gastroesophageal endoscopy
C) Sensory defect D) History
D) Shuffling gait E) Physical exam
E) Lewy bodies
* Most common cause of septic arthritis :
* All are seen in DM type 1 except : A) Intra_Articular injection
A) HLA DR3 and HLA DR4 association B) Trauma
B) Obesity C) Staphylococcus aureus
C) Ketoacidosis D) Osteoarthritis
D) Young age E) Cushing syndrome
E) Autoimmune B cell damage

* Most common symptom of Bronchogenic carcinoma


: * All are causes of chorea except :
A) Chest pain A) PRV
B) Cough B) Rheumatic fever
C) Hemoptysis C) Carbamezipine
D) Dyspnea D) Huntington disease
E) Wheeze and cough E) Wilson disease

* Rate of transmission of HBV by contaminated * All of the following are causes of hemoptysis except
needles is : :
A) 0.3% A) Mitral stenosis
B) 0.03 % B) TB
C) 0.003 % C) PE
D) 3% D) Lung cancer
E) 30% E) Bronchial asthma

* Green house effect of increased temperature by * All of the following are viral disease except :
increase production of : A) Rabies
A) CO2 B) Poliomyelitis
B) O2 C) Yellow fever
C) CO D) Lyme disease
D) N2o E) Dengue fever
E) NO
* All are true about Brucellosis except :
* All of the following are complications of obesity A) Neutrophil Leucocytosis
except : B) Fever of unknown origin
A) Hypothyroidism C) Brucella melitensis is the most virulent
B) DM type 2 D) Weight loss
C) Polycythemia E) Hepatosplenomegaly
D) Obstructive sleep apnea
E) Hyperlipidemia * All are true about Nephritic syndrome except :
A) Oliguria
B) Diastolic HTN

‫ اسامه الخزاعله‬: ‫الدكتور‬ ‫اسئلــــــــة الباطنيــــــة‬


Page 20
C) Lipiduria C) Long QT interval
D) Uremia D) Absent P wave
E) Oedema E) Sine wave

* All are investigation for Thrombophilia except : * All are sign and symptoms of digoxin toxicity
A) Anticardio lipin antibodies except :
B) Factor 5 leiden mutation A) Nausea and vomiting
C) Protein S level B) Confusion
D) Cholesterol level C) Decreased appetite
E) D_dimer D) Hypokalemia
E) Edema
* All are causes of ejection sys murmur except :
A) pregnancy * All are true about complication of MI except :
B) Aortic stenosis A) Ventricular tachycardia
C) Pulmonary stenosis B) Mitral stenosis
D) HOCM C) VSD
E) Mitral stenosis D) Dressler syndrome
E) Heart block
* The physical sign of chest exam , decreased chest
expansion , increased percussion note ( Resonant ) , * The best combination antibiotic for treatment of
decreased breath sound in left side of the lung are aspiration pneumonia is :
mostly : A) Cefuroxime and metronidazole
A) Left side pleural effusion B) Amoxicillin and gentamycin
B) Left side pneumothorax C) Vancomycin and ticaracillin
C) Left side lung fibrosis D) Tetracycline and vancomycin
D) Left side consolidation E) Gentamycin and metronidazole
E) Left side collapse * All are causes of insulin resistance , DM type 2
* Ptosis , large pupils ,eye down and out are except :
characteristic for palsy of : A) Wilson's disease
A) 2 C.N B) Pregnancy
B) 3 C.N C) Cushing disease
C) 4 C.N D) Metabolic syndrome
D) 5 C.N E) Acromegaly
E) 6 C.N
* Short 4th and 5th metacarpals bone is characteristic
* Chvostic sign is seen in one of the following : for :
A) Hypokalemia A) Primary Hyperparathyroidism
B) Hyperkalemia B) Secondary Hyperparathyroidism
C) Hypocalcemia C) Tertiary Hyperparathyroidism
D) Hypercalcemia D) Pseudohypoparathyroidism
E) Hypermagnesemia E) Pseudopseudo_Hyperparathyroidism

* All are causes of finger clubbing except : * All are causes of bloody diarrhea except :
A) Empyema A) Cambylobacter
B) Mesothelioma B) Shigella
C) Celiac disease C) Crohn disease
D) Myocarditis D) E coli
E) Graves disease E) IBS

* All are causes of Postural tremor except : * All are true about ATN except :
A) Parkinsonism A) Urine osmolarity > 500 mosm / L
B) Hyperthyroidism B) Urine Na > 40 mmol / L
C) Alcoholism C) Urine urea / plasma urea < 3:1
D) Anxiety D) Urine creatinine / plasma creatinine < 20:1
E) Pheochromocytoma E) Fractional Na excretion > 2%

* All are true ECG sign for Hyperkalemia except * All are causes of normocytic anemia except :
A) Tall tented T wave A) Chronic blood loss
B) Wide QRS B) Pregnancy

‫ اسامه الخزاعله‬: ‫الدكتور‬ ‫اسئلــــــــة الباطنيــــــة‬


Page 21
C) Hypothyroidism C) Atrial fibrillation
D) Alcoholism D) Asystole
E) Aplastic anemia E) SVT

* All are true about IDA except : * All are true about life threatening attack bronchial
A) Low serum iron asthma except :
B) Low TIBC A) FEV1 < 50% of predicted
C) Low serum ferritin B) Cyanosis
D) Low hemosedrine C) Hypotension and bradycardia
E) High Transferrin D) Normal or high PaCO2
E) PH < 7.30
* All are causes of prolonged PT except :
A) DIC * The first step of management of patient with DKA is
B) Liver disease :
C) Warfarin A) Insulin
D) VWD B) K replacement
E) Factor 10 deficiency C) Fluid replacement normal saline
D) Fluid replacement ringer lactate
* All are true matching about antibiotics and side E) Bicarbonate
effect , except :
A) Doxycycline _ photosensitivity
B) Vancomycin _ Red man syndrome
C) Clindamycin _constipation
D) Chloramphenicol _ Aplastic anemia
E) Metronidazole _-Disulfiram reaction * The first step in management of patient with
* All are sign and symptoms of autonomic neuropathy Hyperkalemia with tall tented T
except : wave :
A) Urine retention A) Calcium Resonium
B) Nocturnal diarrhea B) Dialysis
C) erectile dysfunction C) Calcium gluconate
D) HTN D) Insulin and glucose
E) Horner syndrome E) Nebulized Salbutamol

* All of the following are causes of polyneuropathy * All are cardiovascular complication of acromegaly
except : except :
A) Guillain barre syndrome A) HTN
B) DM B) HOCM
C) Lyme disease C) High output heart failure
D) Charcot Marie tooth syndrome D) Right ventricular hypertrophy
E) Carpal tunnel syndrome E) Coronary artery disease

* All are true about spondyloarthropathy except * All are true about mild acute attack of bronchial
A) Asymmetrical oligo or monoarthritis asthma except :
B) HLA B27 association A) Audible wheeze
C) Extra articular manifestation B) Hyperinflated chest
D) Rheumatoid factor positive C) Hyperresonant percussion
E) Affect spine and sacroiliac joints D) Respiratory acidosis
E) Cough
* All of the following are causes of Nodular
shadowing on chest X_Ray except : * All are causes of type one respiratory failure except
A) Lung hamartoma :
B) Hydatid cyst A) Pneumonia
C) Sarcoidosis B) Pulmonary edema
D) Caplan syndrome C) Obstructive sleep apnea
E) Atelectasis D) Fibrosing alveolitis
E) Emphysema
* All are indication for D/C cardioversion except :
A) Ventricular tachycardia * All are causes of diffuse goitre except :
B) Ventricular fibrillation A) Graves disease

‫ اسامه الخزاعله‬: ‫الدكتور‬ ‫اسئلــــــــة الباطنيــــــة‬


Page 22
B) Hashimoto thyroiditis
C) Plummer disease * Regarding secondary HTN , which is true ?
D) De Quervain thyroiditis A . It account for 90% of the case's .
E) Subacute thyroiditis B . Addison disease is one of the causes .
C . It can be easily controlled .
* All of the following are causes of constipation D . When associated with Hypokalemia ,
except : hyperaldosteronism may be the cause .
A) Hyperkalemia E . All patient should be investigated for secondary
B) Hypercalcemia causes at all ages .
C) Hypothyroidism
D) Iron supplement * A patient is seen in clinic complaining of abdominal
E) Systemic sclerosis pain . Routine blood tests show Na = 142 mmol /L ,
CL = 104 mmol/L , K = 4 mmol/L , HCO3 = 19
* All are therapeutic indication for colonoscopy mmol/L , Ca = 2 mmol/L , PO4 = 1.5 mmol/L , BUN
except : = 28 mg/dl , creatinine = 112 mmol/L , Glucose = 180
A) FAP mg /dl . What is the expected anion gap in mmol/L
B) Hyperplastic polyps and
C) Angiodysplasia osmolarity in mosm/L H2o ?
D) volvulus A . 24 , 314 .
E) Colonic pseudo_obstruction B . 23 , 312 .
C . 32 , 321 .
D . 25 , 315 .
E . 22 , 311 .
* The best screening test for hereditary * 50 years old male patient , with medical history of
Hemochromatosis : DM , free surgical history , social history of infertility
A) Serum iron , abdominal ultrasound showed cirrhotic
B) Serum ferritin liver changes , most likely diagnosis ?
C) Serum Transferrin A . HAV .
D) Serum Transferrin saturation B . Budd Chiari syndrome .
E) Serum TIBC C . Wilson disease .
MED D . Hemochromatosis .
E . Alpha one antitrypsin deficiency .
* A 34 years old female presents with vomiting
preceded by an headache of acute Onest . On exam * Each one of the following may raise ESR except ?
she was conscious , alert with photophobia but no A . Female sex .
neck stiffness . First brain CT scan is reported as B . SLE .
normal . What is the most C . Polycythemia .
appropriate further management ? D . Myeloma .
A . CT brain with contrast . E . Increasing age .
B . Repeat CT brain in 24 hours .
C . CSF examination . * Where is the most common site for primary cardiac
D . Cerebral angiography . tumors to occur in
E . MRI brain . adult ?
A . Left atrium .
* Most common causing microorganism of B . Left ventricle .
spontaneous bacterial peritonitis ? C . Left atrial appendage .
A . E coli . D . Right ventricle .
B . Streptococcus pneumonia . E . Right atrium .
C . Listeria .
D . Staphylococcus aureus . * All of the following can cause Hyperkalemia except
E . Pseudomonas aeruginosa . ?
A . NSAIDs .
* The best treatment of Wernicke Korsakoff B . ACE inhibitors .
syndrome is ? C . Spironolactone .
A . Thiamine . D . Furosemide diuretics .
B . Folate . E . Adrenal insufficiency .
C . Cobaltammine .
D . Pyridoxine . * All of the following are true about Long term
E . Iron . oxygen therapy ( LTOT ) in

‫ اسامه الخزاعله‬: ‫الدكتور‬ ‫اسئلــــــــة الباطنيــــــة‬


Page 23
COPD except ? D . Specific death ratio .
A . The goal is to maintaining SPo2 > 90% and PaO2 E . Case mortality rate .
> 60 mmhg .
B . Main delivery devices include nasal cannula and * Site of action of Spironolactone is ?
venturi mask . A . Proximal convoluted tubules .
C . Does not affect life expectancy in COPD patient . B . Loop of henle ascending part .
D . It is indicated in COPD who have PaO2 < 7.3 C . Distal convoluted tubules .
kpa . D . Loop of henle descending part .
E . To get the benefits of LTOT , patients should E . Juxtaglomerular apparatus .
breath supplemental oxygen for
at least 15 hours per day . * Streptokinase therapy is recommended in acute
pulmonary embolism when ?
* All are risk factors for esophageal cancer except ? A . Patient complains of shortness of breath .
A . GERD . B . Patient complains of chest pain .
B . Achalasia . C . Blood pressure is 70/30 .
C . H pylori . D . Pulmonary angiogram is positive .
D . Smoking . E . Blood tinged sputum .
E . Alcohol .
* All of the following are signs and symptoms of
Acromegaly except ?
* All are causes of Pancytopenia except ? A . Galactorrhea .
A . Paroxysmal nocturnal hemoglobinuria . B . Arthralgia .
B . Systemic lupus erythematosus . C . Excessive sweating .
C . Megaloblastic anemia . D . Sleep apnea .
D . Iron deficiency anemia . E . Hypergonadism .
E . Lymphoma .
* The best combined drugs for management of patient
* In curative splenectomy , best time to give with HTN and history of MI is ?
pneumococcal vaccine ? A . Thiazide and Amlodipine .
A . First week before surgery . B . Thiazide and virapamil .
B . 4 weeks before surgery . C . Atenolol and Ramipril .
C . Immediately after surgery . D . Valsartan and Ramipril .
D . 2 weeks after surgery . E . Thiazide and Ramipril .
E . At least one month after surgery .
* Causes of bilateral hilar lymphadenopathy are all of
* A 25 years old woman complains of shortness of the following except ?
breath after 10 hours of travel by air . RR = 30 BPM , A . Sarcoidosis .
BP = 110/70 , HR = 100 BPM . PE is suspected . The B . Amyloidosis .
optimal duration of warfarin therapy is ? C . Tuberculosis .
A . 6 weeks . D . Lymphoma .
B . 3 months . E . Extrinsic allergic alveolitis
C . 6 months .
D . 1 year . * All of the following are signs and symptoms of
E . Life long . hyperparathyroidism primary type except ?
A . Confusion .
* Woman with family history of cancer is at risk of all B . Anorexia .
of the following except ? C . Polyuria .
A . Endometrial cancer . D . Hypotension .
B . Ovarian cancer . E . Short QT interval .
C . Colon cancer .
D . Gastric cancer . * All of the following are true about pneumocystis
E . Breast cancer . carinii pneumonia except ?
A . It is caused by yeast like fungus pneumocystis
* Numbers of death's with TB in one year over , the jiroveci .
number of population in the middle of the year is B . It is seen in people with cancer , chemotherapy ,
called ? HIV and drug abusers .
A . Mortality rate . C . The risk of pneumocystis carinii pneumonia
B . Crude death rate . increases when CD4 T_cell level
C . Specific death rate . are less than 300 cells / ml .

‫ اسامه الخزاعله‬: ‫الدكتور‬ ‫اسئلــــــــة الباطنيــــــة‬


Page 24
D . X_Ray shows wide spread bilateral opacification * All are causes of warm hemolytic anemia except ?
with low grade fever , non productive cough , loss of A . CLL .
weight and night sweats . B . Lung cancer .
E . In HIV patient drug of choice is co _ trimoxazole . C . SLE .
D . Alpha methyldopa , quinidine , ciprofloxacin .
* Patient with atrial fibrillation , which one of the E . Primaquine , sulfasalazine .
following factors would make a rate control strategy
rather than rhythm control more suitable
A . CHF .
B . Age > 65 years .
C . First presentation .
D . Symptomatic .
E . Atrial fibrillation secondary to a corrected
precipitant . * All of the following are true about extrinsic allergic
alveolitis except ?
* All of the following is considered correct in the A . Caused by hypersensitivity to inhaled organic
management of variceal bleeding except ? dusts .
A . Patient should be resuscitated with IV fluid . B . Type 2 or type 4 hypersensitivity .
B . Monitoring of vital signs is a must . C . Pulmonary function test show restrictive loss of
C . Octreotide agonist should be added if variceal lung function .
bleeding if suspected . D . CBC show significant eosinophilia .
D . There is no need to add antibiotics for the E . Lymphocytic infiltration of alveoli and intra
treatment management GI bleeding caused by alveolar septa on lung biopsy .
esophageal varices .
E . TIPS procedure is an option. * Most common cause of upper GI bleeding is ?
A . Esophageal varices .
* All of the following are true about Mycoplasma B . Gastric ulcer .
pneumonia except ? C . Duodenal ulcer .
A . Autoimmune hemolytic anemia IgM . D . Erosive gastritis .
B . Ceftriaxone is the best treatment . E . Mallory weiss syndrome .
C . Most common primary atypical pneumonia .
D . Bilateral lung consolidation with minimal * Diabetes mellitus diagnostic criteria include 2 hour
symptom . glucose tolerance test , fasting glucose , and HbA1c
E . Walking pneumonia and can cause epidemics respectively ?
A . > 140 mg/dL , > 110 mg/dL , > 6% .
* All of the following are true matching except ? B . >= 200 mg /dL , > 110 mg/dL , > 6.5% .
A . Loud S2 in cor pulmonale and systemic HTN . C . >=200 mg/dL , > =126 mg/dL , >6% .
B . Sof S2 in pulmonary stenosis , aortic stenosis , and D . >=200mg/dL , >=126 mg/dL , >=6.5% .
aortic regurgitation . E . >=180 mg/dL , >=125 mg/dL , >=6.5% .
C. S3 is normal in youth , athletes , pregnants and
common pathological causes * ECG sign of hypothermia is ?
are LVF and mitral regurgitation . A . Tachycardia .
D . Fixed splitting of S1 is heard with ASD . B . U wave .
E . Loud S4 is sign of pathological state always like C . Long QT interval .
LVF , HTN , aortic stenosis . D . Short PR interval .
E . Peaked T wave .
* All are true about immune thrombocytopenic
purpura except ? * All are causes of finger clubbing except ?
A . Peak age 2 _ 6 years old and in adult 20 _ 40 years A . Mesothelioma .
old . B . Crohn disease .
B . Minimal bruising and mucocutaneous bleed . C . Graves disease .
C . Splenomegaly and lymphadenopathy . D . COPD .
D . Bone marrow aspirate shows increased E . Idiopathic pulmonary fibrosis .
Megakaryocytes .
E . Non urgent treatment include corticosteroids , * Which of the following make you suspect acute
intravenous immunoglobulins , splenectomy , rather than chronic failure of kidneys ?
immunomodulators and no role of platelet transfusion A . Anemia .
. B . History of nephrotoxic drug .
C . Small kidneys size .

‫ اسامه الخزاعله‬: ‫الدكتور‬ ‫اسئلــــــــة الباطنيــــــة‬


Page 25
D . Renal osteodystrophy .
E . Peripheral neuropathy . * Which of the following contains the highest calories
contents ?
* All are causes of prerenal azotemia except ? A . Butter .
A . HTN B . Bread .
B . GI bleeding C . Meat .
C . CHF D . Sugar .
D . Sepsis E . Macaroni .
E . Liver cirrhosis

* Iron deficiency anemia can cause all of the


* 25 years old known case of tuberculosis on following except ?
treatment , drop of INR from 3 to 1.2 , which of the A . Koilonychia .
following drugs responsible for this ? B . Onycholysis .
A . Rifampin C . Atrophic glossitis .
B . Isoniazid D . Stomatitis .
C . Ethambutol E . Pica .
D . Pyrazinamide
E . Streptomycin * Causes of Hyperkalemia all of the following except
?
* 25 years old tall man had suddenly rapid and A . Cyclosporine .
shallow breathing . In left CXR found to have 2 cm B . Atenolol .
pneumothorax . What is the best next step ? C . Digoxin .
A . Needle aspiration . D . Triamterene .
B . Chest tube . E . Ethacrynic acid .
C . Intubation .
D . O2 therapy and observation . * All are true about renal tubular acidosis except
E . Thoracotomy . A . Type 1 and type 4 affect distal convoluted tubules
.
* Best time to take simvastatin is ? B . Type 2 affect proximal convoluted tubules .
A . At bed time . C . Type 1 and type 2 associated with Hypokalemia ,
B . Morning . but type 4 associated with Hyperkalemia .
C . Before each meal . D . Nephrocalcinosis associated with type 1 , Fanconi
D . In the evening . syndrome associated with type 2 , Renal insufficiency
E . After each meal. associated with type 4 .
E . Type 4 RTA associated with
* Cyclosporine side effects include all of the hyperaldosteronism .
following except ?
A . Gingival hyperplasia . * A patient has a potassium level of 7 mmol/L .
B . Alopecia . Which of the following
C . Hyperglycemia . intervention is priority ?
D . Jaundice . A . Administer a laxative .
E . Tremor . B . Administer a loop diuretics Furosemide .
C . Administer Spironolactone .
* All are true about cystic fibrosis except ? D . Prepare the patient for dialysis and place the
A . Cor pulmonale patient on cardiac monitor .
B . Rectal prolapse E . Give patient insulin and glucose .
C . Abdominal aortic aneurysm
D . Metabolic Alkalosis * Incidence rate is defined as ?
E . Gallstones A . Number of old case's in a population per unit of
person _ time .
* Activated charcoal is used in one of the following B . Number of new case's in a population per unit
poisoning ? of person _ time .
A . Aminophylline . C . Number of all case's in a population over a defined
B . Lead . period of time .
C . Iron . D . Number of old and new case's in a population per
D . Ethylene glycol . unit of person _ time .
E . Lithium .

‫ اسامه الخزاعله‬: ‫الدكتور‬ ‫اسئلــــــــة الباطنيــــــة‬


Page 26
E . Number of new case's in a population per unit of 3- It is composed of three strains of influenza virus
person over a defined period of one year . 4- It is contraindication in immunocoprpmised
patients
* All of the following are Quantitative tests 5- It is safe in pregnancy
hypothesis except ?
A . Why ?
B . What ?
C . How much ? * Which of the following is ture concerning sickle cell
D . How many ? trait:
E . When ? 1- It is a cause of anemia
* All of the following are criteria for major depression 2- It protects against malatia
except ? 3- It occurs mainly in female
A . Decreased mood . 4- It is a cause of frequent sikle cells in the peripheral
B . Decreased pleasure . blood
C . Hypersomnia . 5- It is associated with hemoglobin H
D . Hallucinations .
E . Suicidal ideation . * Male was admitted with pneumonia caused by
mycoplasma,all of the following are
* A 70 year old woman with a few months history of characteristics of this type of pneumonia except:
leg pain ,gum bleeding and petechiae on the arm after 1- Insidious onset of the disease
the application of a sphygmomanomerter.which of the 2- Fever of 38 C
following vitamin defficiencies dose pateint have ? 3- Sever leukocytosis
1- Vit A 4- Good response to macrolides
2- Vit B 5- Hemolytic anemia
3- Vit C
4- Vit B 12 * All of the following are considered anti-
5- Vit E pseudomanal antibiotics except:
1- Ceftazidime
* Regarding detection of H-pylori infection,the least 2- Ciprofloxacin
useful test is : 3- Ceftriaxone
1- Tissue culture of gastric biopsy 4- Gentamicin
2- Urea breath test 5- Cefipime
3- Histology of gastric biopsy
4- Rapid urease test of gastric biopsy * The activities of school health are except:
5- Stool antigen test 1- Screening of school children to detect and disease
are abnormality
* Which of the following antibiotic has been 2- Teaching students about health system
particularly assciated with hemolytic 3- Providing treatment any student falls sick in the
reactions in patient with G6PD ? school
1- Amoxicillin 4- Health education
2- Clindamycin 5- Immunization
3- Vancomycin
4- Cloxacillin * One of the following is not a systemic complication
5- Trimethoprim/sulfamthoxazole of ulcerative colitis:
1- Erythema nodosum
* Which of the following antibiotics is absolutely 2- Pyoderma gangrenosum
contraindication in pregnant 3- Arthritis
wowen: 4- Malabsorption
1- Imipenem 5- Sclerosing cholangitis
2- Azithromycin
3- Ceftriaxone * Inflammatoy diarrhea is characterized :
4- Cloxacillin 1- Absence of leucocytes in the stool
5- Tetracycillin 2- Absence of fever
3- Presence of leucocytes in the stool
* All the following are true about influenza vaccine 4- Absence of blood in the stool
except:
1- It is given annually * The route of transmission of this virus is fecal oral
2- It is contraindicated in patient with Guillain Barre route:
syndrome 1- Hepatitis B virus

‫ اسامه الخزاعله‬: ‫الدكتور‬ ‫اسئلــــــــة الباطنيــــــة‬


Page 27
2- Hepatitis C virus
3- Hepatitis E virus
4- Hepatitis D virus
5- Human immune deficiency virus

* Which of the following is most sensitive early sign


* In the treatment of infective endocarditis ,all true for detecting diabetic nephropathy:
except: 1- Serum creatinine
1- Persistent fever may indicate a nonbacterial like 2- urinary albumin/creatinine ratio
fungal cause 3- Serum urea level
2- Cidal drugs rather than statics should be used 4- Ranal ultrasonography
3- Always await the culture results before giving 5- Creatinine clearance
any teatment
4- Inrtvenous route is always perferred * The following are clinical features os restrictive
5- Persistent fever may indicate a drug fever cardiomyopathy:
1- Presentation which mimics constrictive pericarditis
* The most common cause of upper gastrointestinal 2- Primarily characterized by impaired diastolic
bleeding: function
1- Esophageal varices 3- Association with primary or secondary amyloidosis
2- GERD 4- Complication of conditions inducing marked
3- Peptic ulcer disease peripheral blood eosinophilla
4- NSAID induced erosive gastritis 5- Gross cardiomwgaly on CXR
5- Esophageal ulcer
* The risk of transmission of hepatitis B virus
* One of the following is not a presentation of liver followingpercutaneous needle stick injury is:
cirrhosis 1- 0.3%
1- Gynecomatia 2- 3%
2- Abdomianl collateral circulation 3- 30%
3- Asterixis 4- 60%
4- Cerebral edema and brain hernation 5- 90%
5- Ascites
* 70 year old man presents for health
* The diagnosis of live cirrhosis is established by: assessment.whichBp reading would indicate
1- Abdominal U/S the highest risk for a cerebrovascular accidents:
2- Prothrombin time and serum albumin 1- 110/90
3- Liver biopsy 2- 120/90
4- Testing for HbsAg ,HCV Ab, ANA, iron and urine 3- 130/70
cupper 4- 140/80
5- Platelets count to todoucumenthypersplenism 5- 155/70

* In the coure of DKA ,which of the following is true * All the following about Hepatitis A infection true
about serum potasstum level: except:
1- It remain unaffected 1- Fecal � oral transmission
2- It can appear normal but total body potassium may
2- It is a common cause of chronic liver disease
actually be low
3- Mortality increases with advancing age
3- It can appear normal but total body potassium may
4- Maximum infectivity of the disease is before the
actually be high
onset of jaundice
4- It will naturally be corrected by insulin
5- Hepatitis A vaccine is protective from the disease
administration
5- None of the above
* In patient with macrocytic anemia,which one the
following features B12
* The most effective method for preventing diabetic
defieciency rather than folate deficiency :
retinopathy is :
1- Mild jaundice
1- Reducing triglyceride level
2- Loss of weigh
2- Controlling blood glucose levels
3- Skin purpura
3- Smoking cessation
4- Lower limb neuropathy
4- Use of an ACE inhibitor
5- Recurrent throat infection
5- Use of aspirin

‫ اسامه الخزاعله‬: ‫الدكتور‬ ‫اسئلــــــــة الباطنيــــــة‬


Page 28
* In hypercapnia are all true except ?
* Peripheral blood neurophil leucocytosis would be an A . Bounding pulse
expected finding in all of B . Headache
the following except: C . Peripheral vasoconstriction
1- Connective tissue diseases D . Confusion
2- Corticosteroid therapt E . Flapping tremor
3- Pregnancy
4- Whooping cough * To assist in the decision whether to hospitalize a
5- Mesenteric infarction patient with community acquired pneumonia , each of
the following may be a factor in favor of
* Regarding santitation barriers all the following are hospitalization except which one ?
true except: A. The patient is confused
1- Protection of water supply B . Serum creatinine > 2.0 mg / dl
2- Protection of food C . Respiratory rate > 30
3- Personal hygiene D . Blood pressure < 90 mg hg
4- Control of flies E . Age > 64 years
5- Using of spring water better than tap water
* All of the following can be used in hypertensive
* All the following are true about Hepatitis B virus crisis except ?
vaccine except: A. Sodium nitroprusside
1- It is given in three doses B . Furosemide
2- Common schedule is: 0, 1 , 6 months C . Labetalol
3- It is safe in immunocompromised individuals D . Clevidipine
4- Booster dose is routinely given after 10 years E . Hydralazine
from finishing the series(the three doses)
5- It is highly protective to hepatitis B virus * Diarrhea may occur with all of the following except
?
* Which of the following features suggests adrenal A . Food poisoning
insufficiency : B . Viral gastroenteritis
1- Hypoglycemia C . Inflammatory bowel disease
2- Hypokalemia D . Colonic Neoplasia
3- Leukopenia E . Hypothyroidism
4- Anemia
5- Hypertension * A 7 years old has severe microcytic anemia due to B
Thalassemia major ( homozygous ) . He requires
* 55 year old asymptomatic obese female presents to frequent blood transfusion( once every sex week's )
your clinic for checkup and To prevent the skeletal and developmental
general advice,youadvice her all of the following complications of thalassemia . Which of the following
condition are strongly medication is also indicated in the treatment of
assciated with obesity except one: patients
1- Insulin resistance requiring frequent blood transfusion ?
2- Hypertension A . Oral calcium supplement
3- Osteoporosis B . Fresh frozen plasma
4- Arteriosclerosis C . Desferrioxamine
5- Hypoventilation D . Pencillamine
E . Cryoprecipitate
* Which of the following is true about Cushing
syndrome ? * One of the following vaccines is a killed vaccine
A. It's due to deficiency of cortisol hormone . A . Measles vaccine
B . Enlarged extremities are commonly seen . B . Mumps vaccine
C . Osteoporosis is not a feature . C . Rubella vaccine
D . Iatrogenic cause is uncommon . D . Pertussis vaccine
E . A moon face and buffalo hump are E . Oral polio vaccine
characteristic .

‫ اسامه الخزاعله‬: ‫الدكتور‬ ‫اسئلــــــــة الباطنيــــــة‬


Page 29
* All of the following are found in patient with C . Tuberculosis
bronchial asthma ? D . Pulmonary embolism
A . Hyperinflated chest E . Goodpasture syndrome
B . Wheezing
C . Dyspnea * Regarding HTN which is true ?
D .Clubbing A . The patient should be screened for secondary HTN
E . Cough .
B . Secondary HTN account for 90% of cases .
* Regarding Sarcoidosis which is true ? C . Addison disease is one of the causes of secondary
A . It is a caseating Granuloma HTN .
B . It has no extrapulmonary manifestation . D . 24 hr urinary collection for metanephrin level
C .Spontaneous resolution may occur in 2 year . may be diagnostic .
D .Steroids is always indicated . E . In isolated systolic HTN no treatment is required .
E . Increase in serum ACE level is diagnostic .
* A 35 year old is evaluated for symptoms of S.O .B .
* All of the following causes exudative pleural Her reports no other lung or heart disease . He smokes
effusion except ? half pack a day for the past 10 years . On examination
A .Constrictive pericarditis his JVP is 2 cm , heart sound normal . Lung clear .
B .Mesothelioma CXR shows hyperinflation and Emphysematous
C .Rheumatoid arthritis changes , while pulmonary function tests shows an
D .Pneumonia FEV1 / FVC ratio < 70% . Which of the following is
E .Tuberculosis most likely diagnosis ?
A . Alpha one antitrypsin deficiency
* Atrial fibrillation can be caused by all the following B . B glucosidase deficiency
except ? C .G6PD
A . Mitral valve disease D .Glucocerebrosidase deficiency
B . HTN E . Growth hormone deficiency
C . Hypothyroidism
D . Pneumonia * Sacroilietis commonly occurs in all of the following
E . Atrial myxoma diseases except ?
A . Ulcerative colitis
* A 73 years old man presents with several episodes B . Crohn disease
of hematemesis . Examination shows signs of C . Gout
orthostatic hypotension and melena . What is the first D . Ankylosing spondylitis
priority in caring for this patient ? E . Psoriatic arthritis
A .NG tube placement and gastric lavage .
B . Resuscitation with adequate IV access and * The following are more in favor of type one DM
appropriate fluid and blood product fusion . than type two except ?
C .IV infusion of H2 receptor antagonist to stop A . Association with ketoacidosis .
bleeding . B . Association with HLA_DR3 or HLA_DR4 .
D .Urgent upper endoscopy . C . Strong family history of diabetes .
E . Urgent surgical consultation . D . Present of islet cell antibodies .
E . Abrupt onset of signs and symptoms .
* Regurgitation AC syndrome one of the following is
true ? * Regarding bronchial asthma all are true except
A . Cardiac enzymes is always elevated . A . It is chronic airway inflammation .
B . ECG changes is mandatory for the diagnosis . B . Treatment with long acting B2 agonist is the
C . Obesity and HTN are non modifiable risk factor . cornerstone .
D . Creatinine kinase can be raised due to other C . PFT increase in FEV1 more than 200 ml and 12 %
non cardiac cause . indicates reversibility .
E . Cardiac troponin remains elevated for 24 hour only D . It's characterized by dyspnea , chest tightness ,
. cough and wheezes .
E . Attack can be induced by exercise
* Signs and symptoms of hypothyroidism include all
the following except ?
A . Thick dry skin
* All of the following can cause Hemoptysis except ? B . Depression
A . Pneumonia C . Constipation
B . Idiopathic pulmonary fibrosis D . Tachycardia

‫ اسامه الخزاعله‬: ‫الدكتور‬ ‫اسئلــــــــة الباطنيــــــة‬


Page 30
E . Weight gain D . Typhi can some times be seen over the affected
joints .
* A 65 years old man develops palpitations and E . Severity is related to skin changes severity .
dizziness . His blood pressure is 80/50 mm hg and his
pulse is regular at 150 / min . His ECG shows a saw * Regarding Nephrotic syndrome all are true except ?
toothed pattern of P waves . Which of the following A . Renal vein thrombosis is a complication .
procedure is most appropriate in converting him back B . Increase bleeding tendency
to sinus rhythm ? C . Increase susceptibility to infection .
A . Carotid sinus pressure D . DM is a recognized cause .
B . Gagging procedure E . Hypoalbuminemia and Proteinuria is present
C . Valsalva maneuver
D . Eyeball compression * 60 years old male presented with massive
E . Electrical cardioversion hematemesis , examination revealed Splenomegaly ,
clubbing and Palmar Erythema . The cause of his
* A 23 years old woman experienced watery diarrhea , hematemesis ?
nausea, vomiting and abdominal cramps 6 hours after A . Mallory Weiss syndrome
eating a salad and a hamburger in a local restaurant . B . Gastric ulcer
The most likely organism causing her disease is ? C . Angiodysplasia
A . Vibrio vulnificus D . Esophageal varices
B . Listeria monocytogenes E . Gastritis
C . Yersinia enterocolitica
D . Clostridium difficile * A 63 years old man is involved in a motor vehicle
E . Staphylococcus aureus accident and is brought to the hospital . On
examination, his blood pressure is 90/60 mm hg ,
* Air conditioner repair worker is more likely to be at pulse 110 / min , and his abdomen is distendes and
risk of contracting ? rigid , he has internal injuries and bleeding on an
A . Hepatitis abdominal CT scan , and requires emergent blood
B . Brucellosis transfusion on his way to the operating room . His
C . Legionnaires disease blood group is AB , which of the following statement
D . Histoplasmosis is not correct ?
E . Tuberculosis A . He is a universal recipient .
B . He has anti A and anti B in his serum .
* A 19 year old woman present with daily acid reflux C . If a cross match is not available, group O RBCs
symptoms and intermittent dysphagia to solid . Since are universal .
onest of symptoms 3 months ago , she has lost 2 kg , D . If insufficient AB blood is available , type A can
she has sporadic rectal bleeding . Her past medical be used .
history is negative . Her father had stomach cancer at E . If insufficient AB blood is available , type B can
age 42 . What would you recommend ? be used .
A . EGD
B . Reassurance * The study of the distribution of a disease or
C . Barium esophagrame condition in a population is
D . PPI therapy for 6_8 week's referred to as ?
E . Abdominal CT A . Descriptive
B . Analytical
* Causes of pancytopenia includes all the following C . Experimental
except ? D . Prospective
A . Megaloblastic anemia E . Retrospective
B . Aplastic anemia
C . Myelofibrosis
D . IDA
E . Acute leukemia * All about irritable bowel syndrome are true except ?
* Which of the following is true of psoriatic A . Abdominal pain relieved by defecation .
arthropathy ? B . Symptoms can be increased by stress .
A . Nail pitting may give a clue to the diagnosis . C . Waking at night with pain or diarrhea support
B . Absence of plaques of psoriasis excludes the the diagnosis .
diagnosis . D . Altered bowel habit is common .
C . The diagnosis can be confirmed by a blood marker E . Examination is often normal .
.
* Regarding diabetic ketoacidosis which is true ?

‫ اسامه الخزاعله‬: ‫الدكتور‬ ‫اسئلــــــــة الباطنيــــــة‬


Page 31
A . Dehydration is less life threatening than treatment . On examination her blood pressure is
hyperglycemia . 120/80 mm hg , pulse 100 / min , and heart and lungs
B . Potassium level initially is high and falls down are normal . There are no clinical sings of DVT .
with treatment . Which of the following
C . Is a late complication of type one DM . investigation is most likely to rule out a PE ?
D . Can be managed as outpatient in most of the cases A . Normal CXR
. B . Normal ECG
E . Elevated WBC count always indicates infection . C . Normal ventilation _ perfusion lung scan .
D . Normal high resolution CT
* All of the following are side effects of steroids E . Normal MRI
except ?
A . Elevated blood pressure * A 66 year old man has progressive shortness of
B . Acne breath due to COPD . He is currently able to do his
C . Skin bruises activities of daily living , but has trouble walking
D . Hypoglycemia more than 50 m . His physical examination reveals
E . Cataract hyperinflation , increased
resonance to percussion , and bilateral expiratory
* Blood gas analysis in type one respiratory failure wheezes . He is in appropriate medical therapy for his
shows ? stage of COPD . Which of the following is
A . High PCO2 and normal PO2 . also indicated in the management of this condition ?
B . Low PCO2 and normal PO2 . A . Meningococcal vaccine
C . Normal PCO2 and high PO2 . B . Yearly influenza vaccine
D . High PCO2 and low PO2 . C . Weight reduction obese
E . Normal PCO2 and low PO2 . D . Haemophilus influenza B vaccination
E . Pneumococcal vaccination
* In salmonellosis which is true ?
A . Positive blood culture occur in the second week . * A 39 year old woman with HTN , is taking
B . Positive stool culture occur in the first week . hydrochlorothiazide ,enalapril and diltiazem . She
C . Splenomegaly doesn't occur . desires to become pregnant . The most important step
D . Can be complicated by Osteomyelitis . would be to ?
E . The route of transmission is air borne . A . Switch HCTZ to Furosemide
B . Switch diltiazem to amlodipine
* One of the following is true about COPD ? C . Discontinue diltiazem
It is one of the cause restrictive PFT . D . Discontinue enalapril
Steroids are the cornerstone treatment . E . Discontinue HCTZ
It is a curable disease .
PFT reversibility always present . * Regarding Acromegaly all are true except ?
Can cause right sided heart failure . A . Caused by tumor in adrenal gland in more than
90 % .
* All of the following are cause of chronic cough B . Excessive sweating .
except ? C . Large hand and feet .
A . GERD D . Increased growth hormone .
B . Sodium valproate E . Proganthism .
C . ACE_i
D . Long term smoking
E . Postnasal drip * All are features of LVF except ?
* Which of the following antiarrhythmic drug A . Orthopnea
mediates it's effect by interfering with movement of B . Hepatomegaly
calcium through the slow channel ? C . Pulsus alternance
A . Phenytoin D . Bilateral basal crepitation
B . Verapamil E . Kerly B line on CXR
C . Lidocaine
D . Amiodarone All of the following statement are correct about
E . Bretylium routine urine examination except ?
A . Specific gravity of 1015 is normal .
* 63 years old woman is seen in the emergency room B . RBCs is a normal finding .
with acute shortness of breath . There is no history of C . One RBC is a normal finding.
heart or lung problems in the past . She was recently D . One WBC is a normal finding.
diagnosed with breast cancer and is under going active E . Negative sugar is a normal finding

‫ اسامه الخزاعله‬: ‫الدكتور‬ ‫اسئلــــــــة الباطنيــــــة‬


Page 32
D . Occupation
* Which oxygen delivery device should be used in E . Residency
COPD patient ?
A . Trauma mask * One of the following disease is considered
B . Nasal cannulae eradication worldwide since more than 30 years
C . Simple face mask A . Smallpox
D . Venturi mask B . Chickenpox
E . Nonrebreather mask C . Plague
D . Typhus
* One of the following can cause Hypokalemia ? E . Mumps
A . Spironolactone
B . Hyperaldosteronism * All of the following sentences are true about the
C . ACE_i assessment of bleeding except ?
D . Addison disease A . Taking blood sample at the time of bleeding
E . Renal tubular acidosis type 4 give an accurate level of HB .
B . Hypovolemia increases the risk of tissue hypoxia
* Which of the following most likely to cause and multiorgan failure .
deviation of trachea ? C. Blood loss could be estimated from the size of
A .Previous TB of the right upper lobe blood clot and the weight of
B .Small right pleural effusion swabs use .
C .A Left basal pneumonia D . Any thirsty patient should be considered
D . Idiopathic pulmonary fibrosis Hypovolemic .
E . Blunted right costophrenic angle E . Arterial blood gas is a quick method to measure
Hb level .
* Optimal medical therapy in patient with carotid
artery stenosis ? * Epidemiology mean ?
A . Smoking cessation A . Study of population ( study of the distribution
B . Antiplatelet agent and determination health related states or even .
C . Control HTN B . Study of disease only .
D . Statin drugs C . Study population growth .
E . All are true D . Study of mortality .
E . Study of chronic disease .
* Skin cancer result from exposure to one of the
following ? * Patient with cystic fibrosis expected to have
A . Infrared rays deficiency in all of the following except ?
B . X rays A . Vitamin A
C . Solar ultraviolet B . Vitamin B12
D . Ionizing radiation C . Vitamin K
E . Microwave D . Vitamin D
E . Vitamin E

* Regarding Mycoplasma pneumonia all are true


except ?
* Regarding immunoglobulins all are true except? A . It is one of atypical pneumonia .
A . Rhesus antibodies are of IgG class . B . Can cause autoimmune Hemolytic anemia .
B . IgG is the first response to invading organism C . Can cause meningioencephalitis .
C . IgA has the highest concentration in human D . Cephalosporin antibiotic is a drug of choice .
secretion . E . Bilateral patchy consolidation .
D . IgM is the best complement activator .
E . IgE mediates allergy by binding antigen on the * One of the following is most suited for detection of
membrane of the cell . diabetic ?
A . Renal ultrasound
* Social class can be measured by all of the following B . Urine analysis for cast
except ? C . Urine albumin
A . Sex D . IVP
B . Income E . Serum creatinine
C . Education

‫ اسامه الخزاعله‬: ‫الدكتور‬ ‫اسئلــــــــة الباطنيــــــة‬


Page 33
* A 30 year old woman is noted to have blood A . 24 hour esophageal PH monitoring .
pressures in the 160/100 mmHg range . She also has B . Chest radiograph .
increased obesity , especially around her abdomen , C . Initiation of omeprazole .
with striae . She has been bruising very easily and has D . Short course of oral corticosteroids .
hirsutism . Which of the following is the most likely E . Initiation of allergy desensitization
diagnosis ?
A . Hyperthyroidism * A patient with known asthma undergoing therapy
B . Hyperparathyroidism with inhaled corticosteroids and intermittent short
C . Coarctation of aorta acting B2 agonist presents with complaints of
D . Cushing syndrome nocturnal awakening secondary to cough and
E . Pheochromocytoma occasional wheezing . This episode occurs three to
four times per week . Pulmonary function tests in the
* A 45 year old man with type two diabetes is noted to past have shown mild obstructive lung disease .
have blood pressures of 145/90 and 150/96 mmHg on Which of the following is the best next step ?
two separate occasions . Which of the following is the Oral steroids
best initial therapy for this patient ? Leukotriene inhibitors
A . Hydrochlorothiazide . Long acting B2 agonist
B . ACE_i . Theophylline
C . B blockers . Antireflux therapy
D . CCB .
E . B blockers and Hydrochlorothiazide . * Which of the following is most accurate ?
. Cough caused by Captopril may resolve with
* A 62 year old diabetic man underwent an abdominal switching to enalapril .
aortic aneurysm repair 2 days ago . He is being treated . Initial treatment of a chronic cough should include
with Gentamycin for a UTI . His urine output has codeine or a similar opiate derivative to suppress the
fallen to 300 ml over 24 hours , and his serum cough .
creatinine has risen from 1.1 mg/dl on admission to . Cough caused by reflux can be effectively ruled
1.9 mg/dl . Which of the following laboratory values out by a negative history of heartburn or dyspepsia
would be most consistent with a prerenal etiology of .
his renal insufficiency ? . More than one condition often is responsible for
A . FEna of 3% . causing a chronic cough in a given patient .
B . Urinary sodium level of 10 mEq/L .
C .CVP reading of 10 mmHg .
D . Gentamycin trough level of 4uq/ml .

* A 55 year old man is noted to have moderately


severe congestive heart
failure with impaired systolic function . Which of the
* A 22 year old woman presents with fatigue, following drugs would
arthralgias , and a nagging dry cough for the past 6 most likely lower his risk of mortality ?
weeks , but no shortness of breath . On physical ACE_i
examination, her lungs are clear to auscultation , and Loop diuretics
she has bilateral pretibial tender Erythematous raised Digoxin
nodules . Which of the following is your Aspirin
best next step ?
A . Chest radiograph . * A 35 year old woman is noted to have chest pain
B . CT chest . with exertion , and has been passing out recently . On
C . Empiric treatment for Postnasal drip . examination she is noted to have a harsh systolic
D . Antinuclear antibody . murmur . Which of the following is the best therapy
E . Initiation of antituberculosis therapy . for her condition ?
Coronary artery bypass
* An obese 50 year old man with a history of asthma Angioplasty
returns with complaints of occasional dyspepsia and Valve replacement
nocturnal cough . He wakes up in the morning with a Carotid endarterectomy
sour taste in his mouth . His current medications
include inhaled corticosteroids and a short acting B2 * A 55 year old man is noted to have congestive heart
agonist . Which of the following should be your next failure and states that he is comfortable at rest but
step ? becomes dyspneic even with walking to bathroom .

‫ اسامه الخزاعله‬: ‫الدكتور‬ ‫اسئلــــــــة الباطنيــــــة‬


Page 34
On echocardiography , he is noted to have an ejection
fraction of 47% . Which of the following is the more * Which of the following patients is most likely to be
accurate description of this patients condition ? a candidate for bone mineral density screening ?
Diastolic dysfunction A 65 year old, thin , white woman who smokes and
Systolic dysfunction is 15 years postmenopausal .
Dilated cardiomyopathy B 40 year old white woman who exercises daily and
Pericardial disease still menstruates .
C healthy 75 year old white man who is sedentary .
* A 24 year old man develops seizures following an D 60 year old overweight black woman .
emergent splenectomy after a car accident . His serum E 35 year old asthmatic woman who took prednisone
sodium level is initially 116 mEq / L and is corrected 40 mg/d for a 2 week course 1 week ago
to 120 mEq/L over the next 3 hours with hypertonic
saline . Which of the following factors most likely led * During which of the following periods in a woman's
to his Hyponatremia ? life is the most bone mass accumulated
Elevation of serum vasopressin Ages 15 to 25
Administration of hypertonic solutions Ages 25 to 35
Volume depletion Ages 35 to 45
Seizures induced Hyponatremia Ages 45 to 55

* A 56 year old man complaining of fatigue and * A 60 year old woman presents with the results of
weight loss . He has never had any health problems , her DEXA scan . She has a T score of _1.5 at the hip
but he has smoked a pack of cigarettes per day for and _2.5 at the spine . Which of the following is the
about a 35 years . skin hyperpigmentation , and digital most accurate interpretation of these results ?
clubbing . He appears euvolemic . The sodium level is She has osteoporosis at the spine and osteopenia at
126 mEq/L , potassium level is 6.7 mEq/L , creatinine the hip .
level is normal, and bicarbonate and chloride levels She has osteoporosis in both area's .
are low . Which of the following is the likely cause of This is a normal examination .
his Hyponatremia given his presentation She has osteoporosis of the hip and osteopenia at the
SIADH spine .
Hypothyroidism You need to know the Z score .
Gastrointestinal losses
Adrenal insufficiency * You see a 70 year old woman in your office for a
Renal insufficiency routine checkup , and you order a DEXA scan for
* An 83 year old woman comes to your office bone mineral density screening . The T score returns
complaining of a headache and mild confusion . Her as 2.5 standard deviations SD in the spine and _2.6 in
medical history is remarkable only for HTN , which is the hip . Which of the following statements is most
well controlled by Hydrochlorothiazide . Her accurate ?
examination and laboratory tests show no signs of This patient has osteopenia .
infection , but her serum sodium level is 119 mEq/L , Estrogen replacement therapy should be started with
and plasma osmolarity is 245 mOsm/kg . She appears an anticipated rebuilding of
to be clinically Hypovolemic . Which of the following bone mass to near normal within 1 year .
is the best initial therapy ? Swimming will help build bone mass .
Fluid restriction Bisphosphonates would reduce the risk of hip fracture
Infusion of 0.9%saline by 50%
Infusion of 3% saline
Infusion of 3% saline with Furosemide * A 35 year old woman complains of calf tenderness
and acute dyspnea . The arterial blood gas reveals
* A 58 year old man has undergone a lengthy colon PO2 ( Partial pressure of oxygen ) of 76 mmHg .
cancer surgery . On the first postoperative day , he is Which of the following is the most common physical
noted to have significant Hyponatremia with a sodium examination finding of pulmonary embolism ?
level of 128 mEq /L . You suspect that the Wheezing .
Hyponatremia is due to the Increased pulmonary component of the second heart
intravenous infusion of hypotonic solution . Which of sound .
the following laboratory findings supports your Tachypnoea .
diagnosis ? Calf swelling .
Urine sodium > mmol/L Pulmonary rales .
Urine osmolality > 200 mOsm /L
Serum osmolarity < 280 mOsm /kg * A 39 year old man is noted to have a DVT without
Serum potassium > 5 mEq/L any known risk factors . He notes that his brother also

‫ اسامه الخزاعله‬: ‫الدكتور‬ ‫اسئلــــــــة الباطنيــــــة‬


Page 35
developed a pulmonary embolism at age 45 years , examination , maculopapular and pustular skin lesions
and his mother developed a clot in the leg when she are noted on the trunk and
was in her thirties . Which of the following is the most extremities . He denies any symptoms of
likely inherited disorder in this patient ? genitourinary tract infection . Synovial fluid analysis
Protein S deficiency is most likely to show which of the following ?
Antithrombin 3 deficiency WBCs 75.000/mm3 with 95% polymorphonuclear
Factor 5 Leiden mutation leukocytes .
Antiphospholipid antibody syndrome RBCs 100.000/mm3 , WBCs 1000/mm3 .
Familial malignancy syndrome WBCs 48.000/mm3 with 80% lymphocytes .
WBCs 500/mm3 with 25% polymorphonuclear
* A 54 year old woman is noted to have cervical leukocytes .
cancer and presents with significant vaginal bleeding
with a hemoglobin level of 7 g/dL . Her left leg is * A 22 year old man presents with complaints of low
swollen , which on Doppler investigation reveals a back pain for 3 to 4 months and stiffness of the
DVT . Which of the following is the best treatment for lumbar area , which worsen with inactivity . He
the thrombus ? reports difficulty in getting out of bed in the morning
Intravenous unfractionated heparin . and may have to roll out sideways , trying not to flex
Fractionated subcutaneous heparin . or rotate the spine to minimize pain . A Lumbosacral
Subcutaneous unfractionated heparin. spine X_Ray film would most likely show which of
Oral warfarin ( Coumadin ) . the following ?
Vena cava filter . Degenerative joint disease with spur information .
Sacroiliitis with increased sclerosis around the
sacroiliac joints .
Vertebral body destruction with wedge fractures .
* A 72 year old man develops severe pain and Osteoporosis with compression fractures of L3_L5 .
swelling in both knees , shortly after undergoing an Diffuse osteonecrosis of the LS spine .
abdominal hernia repair surgery . Physical
examination shows warmth and swelling of both * A 36 year old woman was seen by her physician due
knees with large effusions . Arthrocentesis of the to pain in her hands, wrists , and knees . She is
right knee reveals the presence of intracellular and diagnosed with rheumatoid arthritis . Which of the
extracellular weakly positive birefringent crystals in following treatments will reduce joint inflammation
the synovial fluid . Gram stain is negative . Which of and slow progression of the disease ?
the following is the most likely diagnosis ? NSAIDs
Gout Joint aspiration
Septic arthritis Methotrexate
Calcium oxalate deposition disease Systemic corticosteroids
Reactive arthritis
Pseudogout * An 18 year old adolescent female is brought to the
emergency room because she fainted at a rock concert
* A 65 year old man with a history of chronic HTN , . She apparently recovered spontaneously , did not
DM , and degenerative joint disease presents with exhibit any seizure activity , and has no medical
acute onest of severe pain of the metatarsophalangeal history . Her heart rate is 90 bpm and blood pressure
joint and swelling of the left first toe . Physical 110/70 mmHg . Neurologic examination is normal .
examination shows exquisite tenderness of the joint , The pregnancy test is negative . Which of the
with swelling , warmth , following is the most appropriate management ?
and Erythema . The patient has no history of trauma Admit to hospital for cardiac evaluation .
or other significant medical problems . Synovial fluid Outpatient echocardiogram .
analysis and aspiration is most likely to show which Twenty four hour Holter monitor .
of the following ? Reassurance and discharge home .
Hemorrhagic fluid .
Needle_ shaped , negatively birefringent crystals * A 67 year old woman has diabetes and mild HTN .
Gram negative organisms . She is noted to have some diabetic retinopathy , and
Noninflammatory fluid . she states that she cannot feel her legs . She has
Rhomboidal , positively birefringent crystals recurrent episodes of lightheadedness when she gets
up in the morning .She comes in now because she had
* A 17 year old sexually active adolescent male fainted this morning . Which of the following is the
presents with a 5 day history of fever , chills , and most likely cause of her syncope ?
persistent left ankle pain and swelling . On physical Carotid sinus Hypersinsivety
PE

‫ اسامه الخزاعله‬: ‫الدكتور‬ ‫اسئلــــــــة الباطنيــــــة‬


Page 36
Autonomic neuropathy B Combination therapy including INH, Rifampicine
Critical aortic stenosis and pyrazinamide .
C Observation .
* A 74 year old man with no prior medical problems D Induce three sputum samples .
faints while shaving . He has a quick recovery and has
no neurologic deficits . His blood sugar level is * Which of the following tests is the most appropriate
normal , and ECG shows a normal sinus rhythm . to follow for a patient receiving isoniazed and
Which of the following is the most useful diagnostic rifampin for tuberculosis treatment
test of his probable condition ? A Renal function tests
Carotid massage B Liver function tests
Echocardiogram C Slit_lamp examinations
CT scan of head D Amylase and lipase tests
Serial cardiac enzymes

* Which of the following is not a dermatologic


manifestation of Dermatomyositis ?
Calcinosis cutis
* A 49 year old man is admitted to the intensive care Malar rash
unit with a diagnosis of an inferior MI . His heart rate Gottron papules
is 35 bpm and blood pressure 90/50 mmHg . His ECG Heliotrope rash
shows a mobitz type 1 heart block . Which of the
following is the best next step ? * A 19 year old man develops diarrhea ,and 2 weeks
Atropine later experiences gait difficulties and foot tingling ,
Transvenous pacer which of the following problems associated with his
Lidocaine condition ?
Observation A Acute inflammatory demyelinating
polyneuropathy .
* A 42 year old woman is being treated with B Acute stroke .
infliximab for rheumatoid arthritis . After 6 months of C Myasthenia gravis .
therapy , she develops persistent fever , weight loss , D Inflammatory Myopathy .
and night sweats , and tuberculosis is suspected . E Transverse cord myelitis .
Which of the following is most likely location of the
tuberculosis ? * A 34 year old woman presents with fatigable muscle
A Middle and lower lung zones . weakness with climbing stairs or blow drying her hair
B Pleural space . . This is associated with some shortness of breath ,
C Apical segment of the upper lung lobes . which improves with test . Which of the following
D Cervical or supraclavicular lymph nodes . condition associated with her symptoms ?
A Acute inflammatory demyelinating polyneuropathy
* A 24 year old man has been treated with INH , .
Rifampicine ,and pyrazinamide for active pulmonary B Acute stroke .
tuberculosis . After 3 months , he states that he is C Myasthenia gravis .
having numbness and tingling of both feet but no D Inflammatory Myopathy .
Back pain . He denies taking other medications . E Transverse cord myelitis .
Which of the following is the most appropriate next
step ? * Multiple sclerosis is characterized by which of the
A CT scan of the lumbar spine . following diagnostic findings ?
B Initiate pyridoxine . A Oligoclonal bands in cerebrospinal fluid .
C Continue the tuberculosis agents and monitor for B Increased N_acetyl aspartate with MR spectroscopy
further neurological problems . .
D Initiate a workup for tuberculosis Adenopathy C Abnormal peripheral nerve conduction by
E compression on the femoral nerve. electromyograph ( EMG) / nerve conduction velocity
( NCV) studies .
* A 25 year old woman is seen in the clinic because D Meningeal enhancement on contrast MRI of brain
her father was diagnosed with and has been treated for
tuberculosis . She denies a cough and her chest * A 33 year old man is noted to have exacerbations of
radiograph is normal . A PPD test shows 10 mm of weakness . He is diagnosed with MS . Which of the
induration . Her only medication is an OCP . Which of following findings is consistent with the diagnosis ?
the following is the best next step ? A The diagnosis of MS is based on clinical lesions
A Oral INH and barrier contraception . separated by time and space .

‫ اسامه الخزاعله‬: ‫الدكتور‬ ‫اسئلــــــــة الباطنيــــــة‬


Page 37
B Oligo bands in the CSF are specific for multiple accompanied by roaring tinnitus , aural pressure and
sclerosis . low frequency hearing loss . Her physical examination
C Steroids are effective in improving the course of is normal . What is the most likely diagnosis ?
disease . A Benign Paroxysmal positional vertigo .
D This is a genetic disorder well characterized on B Acute suppurative labyrinthitis .
chromosome 11 C Acute serous labyrinthitis .
D Meniere disease .
E Vertebrobasilar insufficiency .
* A 45 year old woman complains of recurring
episodes of vertigo that began after she bumped her
head rather forcefully 2 week ago . Her spells last 10
* A 33 year old man is noted to have exacerbations of to 15 seconds and occur whenever she goes to bed or
weakness . He is diagnosed with MS . He is noted to wakes up . She denies any hearing loss or tinnitus .
have significantly progressive disease . Which of the Her physical examination is normal except for a latent
following is likely to be helpful for his symptoms of , rotatory nystagmus when she is lying with the right
weakness ear down . What is the most likely diagnosis
A Mitoxantrone A . Acute serous labyrinthitis .
B Corticosteroids therapy B . Benign Paroxysmal positional vertigo .
C Plasmapheresis C . Vestibular migraine .
D Immunoglobulin therapy D . Meniere disease .
E . Viral vestibular neuronitis .
* Which of the following signs is most suggestive of
Parkinson disease rather than the other * A 66 year old woman is noted to have asymptomatic
neurodegenerative diseases ? atrial fibrillation. Which of the following is most
A Unilateral resting tremor . common complication of her atrial fibrillation ?
B Supranuclear downed gaze palsy . A . sudden death .
C Orthostatic hypotension early in the course the B . stroke .
disease . C . shock .
D Early falls . D . dyspnea .
E Abnormal cerebral MRI . E . myocardial infarction

* Which of the following medications is most likely to * A 65 year old woman is presents to the ED with a
be able to help both relieve cardinal features of dyspnea , fatigue , and palpitations . Her blood
Parkinson disease as well as reduce drug induced pressure is 85/50 mmHg and her heart rate is 140 bpm
dyskinesias ? . Which of the following is the best treatment for this
A Levodopa patient ?
B Dopamine agonists A . Diltiazem
C Amantadine B . Metoprolol
D Anticholinergics C . Coumadin
E Haloperidol D . Cardioversion
E . Aspirin
* Which of the following medications would be most
likely to cause drug induced Parkinsonism ? * A 62 year old man is seen in the ED for lower
A Trihexyphenidyl abdominal pain which has since resolved , however ,
B Metoclopramide his heart rate was 80 bpm and palpated irregularly .
C Diazepam On ECG , he is diagnosed with atrial fibrillation with
D Carbidopa a ventricular response of 114 bpm . He does not recall
E Levodopa being told about this condition . Which of the
following is the best initial treatment for this patient ?
* Which of the following tests is used to diagnose A . Diltiazem
benign Paroxysmal positional vertigo ? B . Cardioversion
A Weber test C . Synthroid
B Rinne test D . Ibutilide
C Dix_Hallpike maneuver E . Aspirin
D Brandt Daroff maneuver
E Epley maneuver * Approximately what percentage of patients with
bacterial meningitis present with the classic triad of
* A 40 year old woman has recurring episodes of fever , neck stiffness and altered mental status ?
disabling vertigo , lasting 30 minutes , and A < 25 %

‫ اسامه الخزاعله‬: ‫الدكتور‬ ‫اسئلــــــــة الباطنيــــــة‬


Page 38
B < 50 %
C > 75 %
D > 99 % * A 55 year old man is found to have a serum calcium
of 13 mg/dl after coming to clinic complaining of
* A 58 year old man presents to his physician's office fatigue and thirst for the past month . A chest
complaining of 2 hours of substernal chest pain and radiograph demonstrates a 4 cm mass in the right
dyspnea . Which of the following is the most lower lobe . Which of the following serum tests is
important next step in management ? most likely to reveal the cause of his hypercalcemia ?
A . Administration of Propranolol . A . ACTH .
B . Aspirin to chew . B . ADH .
C . Sublingual nitroglycerin . C . Insulin like growth factor .
D . Administration of a diuretic agent . D . PTH .
E . Chest radiograph . E . PTH related protein .

* A 45 year old man is seen in ED 3 hours of * All of the following conditions may be associated
substernal chest pain radiating to his left arm . The with a thymoma except ?
ECG shows only nonspecific changes . Hearing that A . Erythrocytosis .
the ECG is normal , he requests to go home . Which B .Hypogammaglobulinemia .
of the following statements is most accurate ? C . Myasthenia gravis .
A . The patient may be safely discharged home . D . Polymyositis .
B . If a repeat ECG in 30 minutes is normal , MI is E . Pure red blood cell aplasia .
essentially ruled out and the patient may be safely
discharged . * All of the following statements regarding the anemia
C . The patient should be advised that half of heart of chronic kidney disease are true except
attack patients have a nondiagnostic ECG and A . The degree of anemia correlates with the stage of
serial cardiac biomarkers levels should be assessed chronic kidney disease .
. B . Erythropoietin levels are reduced .
D . The patient should undergo an immediate thallium C . Ferritin is reduced .
stress test to further D . It is typically Normocytic and normochromic .
assesses for coronary artery disease to help clarify the E . Reticulocytes are decreased .
management .
* A 24 year old man with a history of poorly treated
* Which of the following tumor markers is chronic ulcerative colitis is found to have anemia with
appropriately matched with the cell type cancer and a hemoglobin of 9 g/dl and a reduced mean
can be followed during treatment as an adjunct to corpuscular volume . His Ferritin is 250 . Which of
assess disease burden ? the following is the most likely cause of his anemia ?
A CA_125 _ Colon cancer . A Folate deficiency
B Calcitonin _ Follicular carcinoma of the thyroid B Hemoglobinopathy
C CD30 _ Hairy cell leukemia . C Inflammation
D HCG _ GTD . D Iron deficiency
E Neuron specific enolase _ Non small cell carcinoma E Sideroblastic anemia
of the lung
* All of the following statements are true regarding
* A 66 year old woman has CLL with a stable white the criteria to diagnose hypereosinophilic syndrome
blood cell count of between 60.000 and 70.000 /uL . except ?
She is currently hospitalized with pneumococcal A . Increased bone marrow Eosinophils must be
pneumonia . This is the patients third episode of demonstrated .
pneumonia within the past 12 months . What finding B. It is not necessary to have increased circulating
on laboratory testing would be most likely in this Eosinophils .
patient ? C . Primary myeloid leukemia must be excluded .
A Granulocytopenia . D . Reactive Eosinophilia must be excluded .
B Hypogammaglobulinemia . E . There must be less than 20% myeloblasts in blood
C Impaired T cell function with normal T lymphocyte or bone marrow .
count .
D Low CD4 count .
E No specific abnormality is expected .

‫ اسامه الخزاعله‬: ‫الدكتور‬ ‫اسئلــــــــة الباطنيــــــة‬


Page 39
* Which of the following Hemolytic anemias can be A ACTH producing pituitary adenoma .
classified as extracorpuscular ? B Adrenocortical adenoma .
Elliptocytosis C Adrenocortical carcinoma .
PNH D Ectopic ACTH secretion .
Pyruvate kinase deficiency E McCune Albright syndrome .
SCA
TTP * A 27 year old woman with mild obesity is seen in
her primary care office for increased thirst and
* Which of the following statements regarding Polyuria . DM is suspected , and a random plasma
Polycythemia rubra vera is correct ? glucose of 211 mg/d confirms this diagnosis . Which
A . And elevated plasma Erythropoietin level excludes of the following tests will strongly indicate that she
the diagnosis . has type one DM ?
B. Transformation to acute leukemia is common . A .Anti GAD 65 antibody .
C . Thrombocytosis correlates strongly with B .Peroxisome proliferator activated receptor _ 2
thrombotic risk . polymorphism testing .
D . Aspirin should be prescribed to all of these C . Plasma insulin level .
patients to reduce thrombotic risk . D . Testing for HLA_DR3 .
E . Phlebotomy is used only after hydroxyurea and E . There is no laboratory test indicating type one
interferon have been tried . DM .

* All of the following are vitamin K dependent


coagulation factors except ?
A Factor 10
B Factor 7
C Protein C
D Protein S
E Factor 8

* All of the following laboratory values are consistent


with an intravascular Hemolytic anemia except ?
A Increased haptoglobin
B Increased LDH
C Increased Reticulocyte count
D Increased unconjugated bilirubin
E Increased urine hemosiderin

* All of the following are consistent with


nonproliferative diabetic retinopathy except ?
A Blot hemorrhages
B Cotton wool spots
C Neovascularization
D Occurs in first or second decade of DM
E Retinal vascular microaneurysms

* All of the following are potential causes of elevated


LDL except ?
A Anorexia nervosa
B Cirrhosis
C Cholestasis
D Nephrotic syndrome
E Thiazide diuretics

* Which of the following is the most common cause


of Cushing syndrome ?

‫ اسامه الخزاعله‬: ‫الدكتور‬ ‫اسئلــــــــة الباطنيــــــة‬


Page 40

Potrebbero piacerti anche